Você está na página 1de 69

1.

The nurse In-charge in labor and delivery unit administered a dose of terbutaline to a
client without checking the clients pulse. The standard that would be used to determine if
the nurse was negligent is:

a. The physicians orders.
b. The action of a clinical nurse specialist who is recognized expert in the field.
c. The statement in the drug literature about administration of terbutaline.
d. The actions of a reasonably prudent nurse with similar education and experience.


2. Nurse Trish is caring for a female client with a history of GI bleeding, sickle cell disease,
and a platelet count of 22,000/l. The female client is dehydrated and receiving dextrose
5% in half-normal saline solution at 150 ml/hr. The client complains of severe bone pain
and is scheduled to receive a dose of morphine sulfate. In administering the medication,
Nurse Trish should avoid which route?
a. I.V
b. I.M
c. Oral
d. S.C


3. Dr. Garcia writes the following order for the client who has been recently admitted
Digoxin .125 mg P.O. once daily. To prevent a dosage error, how should the nurse
document this order onto the medication administration record?

a. Digoxin .1250 mg P.O. once daily
b. Digoxin 0.1250 mg P.O. once daily
c. Digoxin 0.125 mg P.O. once daily
d. Digoxin .125 mg P.O. once daily


4. A newly admitted female client was diagnosed with deep vein thrombosis. Which nursing
diagnosis should receive the highest priority?

a. Ineffective peripheral tissue perfusion related to venous congestion.
b. Risk for injury related to edema.
c. Excess fluid volume related to peripheral vascular disease.
d. Impaired gas exchange related to increased blood flow.

5. Nurse Betty is assigned to the following clients. The client that the nurse would see first
after endorsement?
a. A 34 year-old post operative appendectomy client of five hours who is complaining of
pain.
b. A 44 year-old myocardial infarction (MI) client who is complaining of nausea.
c. A 26 year-old client admitted for dehydration whose intravenous (IV) has infiltrated.
d. A 63 year-old post operatives abdominal hysterectomy client of three days whose
incisional dressing is saturated with serosanguinous fluid.


6. Nurse Gail places a client in a four-point restraint following orders from the physician.
The client care plan should include:
a. Assess temperature frequently.
b. Provide diversional activities.
c. Check circulation every 15-30 minutes.
d. Socialize with other patients once a shift.


7. A male client who has severe burns is receiving H2 receptor antagonist therapy. The
nurse In-charge knows the purpose of this therapy is to:
a. Prevent stress ulcer
b. Block prostaglandin synthesis
c. Facilitate protein synthesis.
d. Enhance gas exchange


8. The doctor orders hourly urine output measurement for a postoperative male client. The
nurse Trish records the following amounts of output for 2 consecutive hours: 8 a.m.: 50 ml;
9 a.m.: 60 ml. Based on these amounts, which action should the nurse take?
a. Increase the I.V. fluid infusion rate
b. Irrigate the indwelling urinary catheter
c. Notify the physician
d. Continue to monitor and record hourly urine output


9. Tony, a basketball player twist his right ankle while playing on the court and seeks care
for ankle pain and swelling. After the nurse applies ice to the ankle for 30 minutes, which
statement by Tony suggests that ice application has been effective?
a. My ankle looks less swollen now.
b. My ankle feels warm.
c. My ankle appears redder now.
d. I need something stronger for pain relief

10.The physician prescribes a loop diuretic for a client. When administering this drug, the
nurse anticipates that the client may develop which electrolyte imbalance?
a. Hypernatremia
b. Hyperkalemia
c. Hypokalemia
d. Hypervolemia


11.She finds out that some managers have benevolent-authoritative style of management.
Which of the following behaviors will she exhibit most likely?
a. Have condescending trust and confidence in their subordinates.
b. Gives economic and ego awards.
c. Communicates downward to staffs.
d. Allows decision making among subordinates.


12. Nurse Amy is aware that the following is true about functional nursing
a. Provides continuous, coordinated and comprehensive nursing services.
b. One-to-one nurse patient ratio.
c. Emphasize the use of group collaboration.
d. Concentrates on tasks and activities.


13.Which type of medication order might read "Vitamin K 10 mg I.M. daily 3 days?"

a. Single order
b. Standard written order
c. Standing order
d. Stat order


14.A female client with a fecal impaction frequently exhibits which clinical manifestation?
a. Increased appetite
b. Loss of urge to defecate
c. Hard, brown, formed stools
d. Liquid or semi-liquid stools


15.Nurse Linda prepares to perform an otoscopic examination on a female client. For proper
visualization, the nurse should position the client's ear by:

a. Pulling the lobule down and back
b. Pulling the helix up and forward
c. Pulling the helix up and back
d. Pulling the lobule down and forward


16. Which instruction should nurse Tom give to a male client who is having external
radiation therapy:

a. Protect the irritated skin from sunlight.
b. Eat 3 to 4 hours before treatment.
c. Wash the skin over regularly.
d. Apply lotion or oil to the radiated area when it is red or sore.


17.In assisting a female client for immediate surgery, the nurse In-charge is aware that she
should:

a. Encourage the client to void following preoperative medication.
b. Explore the clients fears and anxieties about the surgery.
c. Assist the client in removing dentures and nail polish.
d. Encourage the client to drink water prior to surgery.


18. A male client is admitted and diagnosed with acute pancreatitis after a holiday
celebration of excessive food and alcohol. Which assessment finding reflects this diagnosis?

a. Blood pressure above normal range.
b. Presence of crackles in both lung fields.
c. Hyperactive bowel sounds
d. Sudden onset of continuous epigastric and back pain.


19. Which dietary guidelines are important for nurse Oliver to implement in caring for the
client with burns?

a. Provide high-fiber, high-fat diet
b. Provide high-protein, high-carbohydrate diet.
c. Monitor intake to prevent weight gain.
d. Provide ice chips or water intake.


20.Nurse Hazel will administer a unit of whole blood, which priority information should the
nurse have about the client?

a. Blood pressure and pulse rate.
b. Height and weight.
c. Calcium and potassium levels
d. Hgb and Hct levels.


21. Nurse Michelle witnesses a female client sustain a fall and suspects that the leg may be
broken. The nurse takes which priority action?

a. Takes a set of vital signs.
b. Call the radiology department for X-ray.
c. Reassure the client that everything will be alright.
d. Immobilize the leg before moving the client.


22.A male client is being transferred to the nursing unit for admission after receiving a
radium implant for bladder cancer. The nurse in-charge would take which priority action in
the care of this client?

a. Place client on reverse isolation.
b. Admit the client into a private room.
c. Encourage the client to take frequent rest periods.
d. Encourage family and friends to visit.


23.A newly admitted female client was diagnosed with agranulocytosis. The nurse
formulates which priority nursing diagnosis?

a. Constipation
b. Diarrhea
c. Risk for infection
d. Deficient knowledge


24.A male client is receiving total parenteral nutrition suddenly demonstrates signs and
symptoms of an air embolism. What is the priority action by the nurse?

a. Notify the physician.
b. Place the client on the left side in the Trendelenburg position.
c. Place the client in high-Fowlers position.
d. Stop the total parenteral nutrition.


25.Nurse May attends an educational conference on leadership styles. The nurse is sitting
with a nurse employed at a large trauma center who states that the leadership style at the
trauma center is task-oriented and directive. The nurse determines that the leadership style
used at the trauma center is:

a. Autocratic.
b. Laissez-faire.
c. Democratic.
d. Situational


26.The physician orders DS 500 cc with KCl 10 mEq/liter at 30 cc/hr. The nurse in-charge is
going to hang a 500 cc bag. KCl is supplied 20 mEq/10 cc. How many ccs of KCl will be
added to the IV solution?

a. .5 cc
b. 5 cc
c. 1.5 cc
d. 2.5 cc


27.A child of 10 years old is to receive 400 cc of IV fluid in an 8 hour shift. The IV drip
factor is 60. The IV rate that will deliver this amount is:

a. 50 cc/ hour
b. 55 cc/ hour
c. 24 cc/ hour
d. 66 cc/ hour


28.The nurse is aware that the most important nursing action when a client returns from
surgery is:

a. Assess the IV for type of fluid and rate of flow.
b. Assess the client for presence of pain.
c. Assess the Foley catheter for patency and urine output
d. Assess the dressing for drainage.


29. Which of the following vital sign assessments that may indicate cardiogenic shock after
myocardial infarction?

a. BP 80/60, Pulse 110 irregular
b. BP 90/50, Pulse 50 regular
c. BP 130/80, Pulse 100 regular
d. BP 180/100, Pulse 90 irregular


30.Which is the most appropriate nursing action in obtaining a blood pressure
measurement?

a. Take the proper equipment, place the client in a comfortable position, and record the
appropriate information in the clients chart.
b. Measure the clients arm, if you are not sure of the size of cuff to use.
c. Have the client recline or sit comfortably in a chair with the forearm at the level of the
heart.
d. Document the measurement, which extremity was used, and the position that the client
was in during the measurement.


31.Asking the questions to determine if the person understands the health teaching
provided by the nurse would be included during which step of the nursing process?

a. Assessment
b. Evaluation
c. Implementation
d. Planning and goals


32.Which of the following item is considered the single most important factor in assisting
the health professional in arriving at a diagnosis or determining the persons needs?

a. Diagnostic test results
b. Biographical date
c. History of present illness
d. Physical examination


33.In preventing the development of an external rotation deformity of the hip in a client
who must remain in bed for any period of time, the most appropriate nursing action would
be to use:

a. Trochanter roll extending from the crest of the ileum to the midthigh.
b. Pillows under the lower legs.
c. Footboard
d. Hip-abductor pillow


34.Which stage of pressure ulcer development does the ulcer extend into the subcutaneous
tissue?

a. Stage I
b. Stage II
c. Stage III
d. Stage IV


35.When the method of wound healing is one in which wound edges are not surgically
approximated and integumentary continuity is restored by granulations, the wound healing
is termed

a. Second intention healing
b. Primary intention healing
c. Third intention healing
d. First intention healing


36.An 80-year-old male client is admitted to the hospital with a diagnosis of pneumonia.
Nurse Oliver learns that the client lives alone and hasnt been eating or drinking. When
assessing him for dehydration, nurse Oliver would expect to find:

a. Hypothermia
b. Hypertension
c. Distended neck veins
d. Tachycardia


37.The physician prescribes meperidine (Demerol), 75 mg I.M. every 4 hours as needed, to
control a clients postoperative pain. The package insert is Meperidine, 100 mg/ml. How
many milliliters of meperidine should the
client receive?

a. 0.75
b. 0.6
c. 0.5
d. 0.25


38. A male client with diabetes mellitus is receiving insulin. Which statement correctly
describes an insulin unit?

a. Its a common measurement in the metric system.
b. Its the basis for solids in the avoirdupois system.
c. Its the smallest measurement in the apothecary system.
d. Its a measure of effect, not a standard measure of weight or quantity.


39.Nurse Oliver measures a clients temperature at 102 F. What is the equivalent
Centigrade temperature?

a. 40.1 C
b. 38.9 C
c. 48 C
d. 38 C


40.The nurse is assessing a 48-year-old client who has come to the physicians office for his
annual physical exam. One of the first physical
signs of aging is:

a. Accepting limitations while developing assets.
b. Increasing loss of muscle tone.
c. Failing eyesight, especially close vision.
d. Having more frequent aches and pains.


41.The physician inserts a chest tube into a female client to treat a pneumothorax. The tube
is connected to water-seal drainage. The nurse in-charge can prevent chest tube air leaks
by:

a. Checking and taping all connections.
b. Checking patency of the chest tube.
c. Keeping the head of the bed slightly elevated.
d. Keeping the chest drainage system below the level of the chest.


42.Nurse Trish must verify the clients identity before administering medication. She is
aware that the safest way to verify identity is to:

a. Check the clients identification band.
b. Ask the client to state his name.
c. State the clients name out loud and wait a client to repeat it.
d. Check the room number and the clients name on the bed.


43.The physician orders dextrose 5 % in water, 1,000 ml to be infused over 8 hours. The
I.V. tubing delivers 15 drops/ml. Nurse John should run the I.V. infusion at a rate of:

a. 30 drops/minute
b. 32 drops/minute
c. 20 drops/minute
d. 18 drops/minute


44.If a central venous catheter becomes disconnected accidentally, what should the nurse
in-charge do immediately?

a. Clamp the catheter
b. Call another nurse
c. Call the physician
d. Apply a dry sterile dressing to the site.


45.A female client was recently admitted. She has fever, weight loss, and watery diarrhea is
being admitted to the facility. While assessing the client, Nurse Hazel inspects the clients
abdomen and notice that it is slightly concave. Additional assessment should proceed in
which order:

a. Palpation, auscultation, and percussion.
b. Percussion, palpation, and auscultation.
c. Palpation, percussion, and auscultation.
d. Auscultation, percussion, and palpation.


46. Nurse Betty is assessing tactile fremitus in a client with pneumonia. For this
examination, nurse Betty should use the:

a. Fingertips
b. Finger pads
c. Dorsal surface of the hand
d. Ulnar surface of the hand


47. Which type of evaluation occurs continuously throughout the teaching and learning
process?

a. Summative
b. Informative
c. Formative
d. Retrospective


48.A 45 year old client, has no family history of breast cancer or other risk factors for this
disease. Nurse John should instruct her to have
mammogram how often?

a. Twice per year
b. Once per year
c. Every 2 years
d. Once, to establish baseline


49.A male client has the following arterial blood gas values: pH 7.30; Pao2 89 mmHg;
Paco2 50 mmHg; and HCO3 26mEq/L. Based on these values, Nurse Patricia should expect
which condition?

a. Respiratory acidosis
b. Respiratory alkalosis
c. Metabolic acidosis
d. Metabolic alkalosis


50.Nurse Len refers a female client with terminal cancer to a local hospice. What is the goal
of this referral?

a. To help the client find appropriate treatment options.
b. To provide support for the client and family in coping with terminal illness.
c. To ensure that the client gets counseling regarding health care costs.
d. To teach the client and family about cancer and its treatment.


51.When caring for a male client with a 3-cm stage I pressure ulcer on the coccyx, which of
the following actions can the nurse institute
independently?

a. Massaging the area with an astringent every 2 hours.
b. Applying an antibiotic cream to the area three times per day.
c. Using normal saline solution to clean the ulcer and applying a protective dressing as
necessary.
d. Using a povidone-iodine wash on the ulceration three times per day.


52.Nurse Oliver must apply an elastic bandage to a clients ankle and calf. He should apply
the bandage beginning at the clients:

a. Knee
b. Ankle
c. Lower thigh
d. Foot


53.A 10 year old child with type 1 diabetes develops diabetic ketoacidosis and receives a
continuous insulin infusion. Which condition represents the greatest risk to this child?

a. Hypernatremia
b. Hypokalemia
c. Hyperphosphatemia
d. Hypercalcemia


54.Nurse Len is administering sublingual nitrglycerin (Nitrostat) to the newly admitted
client. Immediately afterward, the client may experience:

a. Throbbing headache or dizziness
b. Nervousness or paresthesia.
c. Drowsiness or blurred vision.
d. Tinnitus or diplopia.


55.Nurse Michelle hears the alarm sound on the telemetry monitor. The nurse quickly looks
at the monitor and notes that a client is in a ventricular tachycardia. The nurse rushes to
the clients room. Upon reaching the clients bedside, the nurse would take which action
first?

a. Prepare for cardioversion
b. Prepare to defibrillate the client
c. Call a code
d. Check the clients level of consciousness


56.Nurse Hazel is preparing to ambulate a female client. The best and the safest position for
the nurse in assisting the client is to stand:

a. On the unaffected side of the client.
b. On the affected side of the client.
c. In front of the client.
d. Behind the client.


57.Nurse Janah is monitoring the ongoing care given to the potential organ donor who has
been diagnosed with brain death. The nurse determines that the standard of care had been
maintained if which of the following data is observed?

a. Urine output: 45 ml/hr
b. Capillary refill: 5 seconds
c. Serum pH: 7.32
d. Blood pressure: 90/48 mmHg


58. Nurse Amy has an order to obtain a urinalysis from a male client with an indwelling
urinary catheter. The nurse avoids which of the following, which contaminate the specimen?

a. Wiping the port with an alcohol swab before inserting the syringe.
b. Aspirating a sample from the port on the drainage bag.
c. Clamping the tubing of the drainage bag.
d. Obtaining the specimen from the urinary drainage bag.


59.Nurse Meredith is in the process of giving a client a bed bath. In the middle of the
procedure, the unit secretary calls the nurse on the intercom to tell the nurse that there is
an emergency phone call. The appropriate nursing action is to:

a. Immediately walk out of the clients room and answer the phone call.
b. Cover the client, place the call light within reach, and answer the phone call.
c. Finish the bed bath before answering the phone call.
d. Leave the clients door open so the client can be monitored and the nurse can answer the
phone call.


60. Nurse Janah is collecting a sputum specimen for culture and sensitivity testing from a
client who has a productive cough. Nurse Janah plans to implement which intervention to
obtain the specimen?

a. Ask the client to expectorate a small amount of sputum into the emesis basin.
b. Ask the client to obtain the specimen after breakfast.
c. Use a sterile plastic container for obtaining the specimen.
d. Provide tissues for expectoration and obtaining the specimen.


61. Nurse Ron is observing a male client using a walker. The nurse determines that the
client is using the walker correctly if the client:

a. Puts all the four points of the walker flat on the floor, puts weight on the hand pieces,
and then walks into it.
b. Puts weight on the hand pieces, moves the walker forward, and then walks into it.
c. Puts weight on the hand pieces, slides the walker forward, and then walks into it.
d. Walks into the walker, puts weight on the hand pieces, and then puts all four points of
the walker flat on the floor.


62.Nurse Amy has documented an entry regarding client care in the clients medical record.
When checking the entry, the nurse realizes that incorrect information was documented.
How does the nurse correct this error?

a. Erases the error and writes in the correct information.
b. Uses correction fluid to cover up the incorrect information and writes in the correct
information.
c. Draws one line to cross out the incorrect information and then initials the change.
d. Covers up the incorrect information completely using a black pen and writes in the
correct information


63.Nurse Ron is assisting with transferring a client from the operating room table to a
stretcher. To provide safety to the client, the nurse should:

a. Moves the client rapidly from the table to the stretcher.
b. Uncovers the client completely before transferring to the stretcher.
c. Secures the client safety belts after transferring to the stretcher.
d. Instructs the client to move self from the table to the stretcher.


64.Nurse Myrna is providing instructions to a nursing assistant assigned to give a bed bath
to a client who is on contact precautions. Nurse Myrna instructs the nursing assistant to use
which of the following protective items when giving bed bath?

a. Gown and goggles
b. Gown and gloves
c. Gloves and shoe protectors
d. Gloves and goggles


65. Nurse Oliver is caring for a client with impaired mobility that occurred as a result of a
stroke. The client has right sided arm and leg weakness. The nurse would suggest that the
client use which of the following assistive devices that would provide the best stability for
ambulating?

a. Crutches
b. Single straight-legged cane
c. Quad cane
d. Walker


66.A male client with a right pleural effusion noted on a chest X-ray is being prepared for
thoracentesis. The client experiences severe dizziness when sitting upright. To provide a
safe environment, the nurse assists the client to which position for the procedure?

a. Prone with head turned toward the side supported by a pillow.
b. Sims position with the head of the bed flat.
c. Right side-lying with the head of the bed elevated 45 degrees.
d. Left side-lying with the head of the bed elevated 45 degrees.


67.Nurse John develops methods for data gathering. Which of the following criteria of a
good instrument refers to the ability of the instrument to yield the same results upon its
repeated administration?

a. Validity
b. Specificity
c. Sensitivity
d. Reliability


68.Harry knows that he has to protect the rights of human research subjects. Which of the
following actions of Harry ensures anonymity?

a. Keep the identities of the subject secret
b. Obtain informed consent
c. Provide equal treatment to all the subjects of the study.
d. Release findings only to the participants of the study


69.Patients refusal to divulge information is a limitation because it is beyond the control of
Tifanny. What type of research is appropriate for this study?

a. Descriptive- correlational
b. Experiment
c. Quasi-experiment
d. Historical


70.Nurse Ronald is aware that the best tool for data gathering is?

a. Interview schedule
b. Questionnaire
c. Use of laboratory data
d. Observation


71.Monica is aware that there are times when only manipulation of study variables is
possible and the elements of control or randomization are not attendant. Which type of
research is referred to this?

a. Field study
b. Quasi-experiment
c. Solomon-Four group design
d. Post-test only design


72.Cherry notes down ideas that were derived from the description of an investigation
written by the person who conducted it. Which type of reference source refers to this?

a. Footnote
b. Bibliography
c. Primary source
d. Endnotes


73.When Nurse Trish is providing care to his patient, she must remember that her duty is
bound not to do doing any action that will cause the patient harm. This is the meaning of
the bioethical principle:

a. Non-maleficence
b. Beneficence
c. Justice
d. Solidarity


74.When a nurse in-charge causes an injury to a female patient and the injury caused
becomes the proof of the negligent act, the presence of the injury is said to exemplify the
principle of:

a. Force majeure
b. Respondeat superior
c. Res ipsa loquitor
d. Holdover doctrine


75.Nurse Myrna is aware that the Board of Nursing has quasi-judicial power. An example of
this power is:

a. The Board can issue rules and regulations that will govern the practice of nursing
b. The Board can investigate violations of the nursing law and code of ethics
c. The Board can visit a school applying for a permit in collaboration with CHED
d. The Board prepares the board examinations


76. When the license of nurse Krina is revoked, it means that she:

a. Is no longer allowed to practice the profession for the rest of her life
b. Will never have her/his license re-issued since it has been revoked
c. May apply for re-issuance of his/her license based on certain conditions stipulated in RA
9173
d. Will remain unable to practice professional nursing


77.Ronald plans to conduct a research on the use of a new method of pain assessment
scale. Which of the following is the second step in the conceptualizing phase of the research
process?

a. Formulating the research hypothesis
b. Review related literature
c. Formulating and delimiting the research problem
d. Design the theoretical and conceptual framework


78. The leader of the study knows that certain patients who are in a specialized research
setting tend to respond psychologically to the conditions of the study. This referred to as :

a. Cause and effect
b. Hawthorne effect
c. Halo effect
d. Horns effect


79.Mary finally decides to use judgment sampling on her research. Which of the following
actions of is correct?

a. Plans to include whoever is there during his study.
b. Determines the different nationality of patients frequently admitted and decides to get
representations samples from each.
c. Assigns numbers for each of the patients, place these in a fishbowl and draw 10 from it.
d. Decides to get 20 samples from the admitted patients


80. The nursing theorist who developed transcultural nursing theory is:

a. Florence Nightingale
b. Madeleine Leininger
c. Albert Moore
d. Sr. Callista Roy


81.Marion is aware that the sampling method that gives equal chance to all units in the
population to get picked is:

a. Random
b. Accidental
c. Quota
d. Judgment


82.John plans to use a Likert Scale to his study to determine the:

a. Degree of agreement and disagreement
b. Compliance to expected standards
c. Level of satisfaction
d. Degree of acceptance


83.Which of the following theory addresses the four modes of adaptation?

a. Madeleine Leininger
b. Sr. Callista Roy
c. Florence Nightingale
d. Jean Watson


84.Ms. Garcia is responsible to the number of personnel reporting to her. This principle
refers to:

a. Span of control
b. Unity of command
c. Downward communication
d. Leader


85.Ensuring that there is an informed consent on the part of the patient before a surgery is
done, illustrates the bioethical principle of:

a. Beneficence
b. Autonomy
c. Veracity
d. Non-maleficence


86.Nurse Reese is teaching a female client with peripheral vascular disease about foot care;
Nurse Reese should include which instruction?

a. Avoid wearing cotton socks.
b. Avoid using a nail clipper to cut toenails.
c. Avoid wearing canvas shoes.
d. Avoid using cornstarch on feet.


87.A client is admitted with multiple pressure ulcers. When developing the client's diet plan,
the nurse should include:

a. Fresh orange slices
b. Steamed broccoli
c. Ice cream
d. Ground beef patties


88.The nurse prepares to administer a cleansing enema. What is the most common client
position used for this procedure?

a. Lithotomy
b. Supine
c. Prone
d. Sims left lateral


89.Nurse Marian is preparing to administer a blood transfusion. Which action should the
nurse take first?

a. Arrange for typing and cross matching of the clients blood.
b. Compare the clients identification wristband with the tag on the unit of blood.
c. Start an I.V. infusion of normal saline solution.
d. Measure the clients vital signs.


90.A 65 years old male client requests his medication at 9 p.m. instead of 10 p.m. so that
he can go to sleep earlier. Which type of nursing intervention is required?

a. Independent
b. Dependent
c. Interdependent
d. Intradependent


91.A female client is to be discharged from an acute care facility after treatment for right
leg thrombophlebitis. The Nurse Betty notes that the client's leg is pain-free, without
redness or edema. The nurse's actions reflect which step of the nursing process?

a. Assessment
b. Diagnosis
c. Implementation
d. Evaluation


92.Nursing care for a female client includes removing elastic stockings once per day. The
Nurse Betty is aware that the rationale for this intervention?

a. To increase blood flow to the heart
b. To observe the lower extremities
c. To allow the leg muscles to stretch and relax
d. To permit veins in the legs to fill with blood.


93.Which nursing intervention takes highest priority when caring for a newly admitted client
who's receiving a blood transfusion?

a. Instructing the client to report any itching, swelling, or dyspnea.
b. Informing the client that the transfusion usually take 1 to 2 hours.
c. Documenting blood administration in the client care record.
d. Assessing the clients vital signs when the transfusion ends.


94.A male client complains of abdominal discomfort and nausea while receiving tube
feedings. Which intervention is most appropriate for this problem?

a. Give the feedings at room temperature.
b. Decrease the rate of feedings and the concentration of the formula.
c. Place the client in semi-Fowler's position while feeding.
d. Change the feeding container every 12 hours.


95.Nurse Patricia is reconstituting a powdered medication in a vial. After adding the solution
to the powder, she nurse should:

a. Do nothing.
b. Invert the vial and let it stand for 3 to 5 minutes.
c. Shake the vial vigorously.
d. Roll the vial gently between the palms.


96.Which intervention should the nurse Trish use when administering oxygen by face mask
to a female client?

a. Secure the elastic band tightly around the client's head.
b. Assist the client to the semi-Fowler position if possible.
c. Apply the face mask from the client's chin up over the nose.
d. Loosen the connectors between the oxygen equipment and humidifier.


97.The maximum transfusion time for a unit of packed red blood cells (RBCs) is:

a. 6 hours
b. 4 hours
c. 3 hours
d. 2 hours


98.Nurse Monique is monitoring the effectiveness of a client's drug therapy. When should
the nurse Monique obtain a blood sample to measure the trough drug level?

a. 1 hour before administering the next dose.
b. Immediately before administering the next dose.
c. Immediately after administering the next dose.
d. 30 minutes after administering the next dose.


99.Nurse May is aware that the main advantage of using a floor stock system is:

a. The nurse can implement medication orders quickly.
b. The nurse receives input from the pharmacist.
c. The system minimizes transcription errors.
d. The system reinforces accurate calculations.


100. Nurse Oliver is assessing a client's abdomen. Which finding should the nurse report as
abnormal?

a. Dullness over the liver.
b. Bowel sounds occurring every 10 seconds.
c. Shifting dullness over the abdomen.
d. Vascular sounds heard over the renal arteries.













1. Answer: (D) The actions of a reasonably prudent nurse with similar education and
experience.
Rationale: The standard of care is determined by the average degree of skill, care, and
diligence by nurses in similar circumstances.

2. Answer: (B) I.M
Rationale: With a platelet count of 22,000/l, the clients tends to bleed easily. Therefore,
the nurse should avoid using the I.M. route because the area is a highly vascular and can
bleed readily when penetrated by a needle. The bleeding can be difficult to stop.

3. Answer: (C) Digoxin 0.125 mg P.O. once daily
Rationale: The nurse should always place a zero before a decimal point so that no one
misreads the figure, which could result in a dosage error. The nurse should never insert a
zero at the end of a dosage that includes a decimal point because this could be misread,
possibly leading to a tenfold increase in the dosage.

4. Answer: (A) Ineffective peripheral tissue perfusion related to venous congestion.
Rationale: Ineffective peripheral tissue perfusion related to venous congestion takes the
highest priority because venous inflammation and clot formation impede blood flow in a
client with deep vein thrombosis.

5. Answer: (B) A 44 year-old myocardial infarction (MI) client who is complaining of nausea.
Rationale: Nausea is a symptom of impending myocardial infarction (MI) and should be
assessed immediately so that treatment can be instituted and further damage to the heart
is avoided.

6. Answer: (C) Check circulation every 15-30 minutes.
Rationale: Restraints encircle the limbs, which place the client at risk for circulation being
restricted to the distal areas of the extremities. Checking the clients circulation every 15-30
minutes will allow the nurse to adjust the restraints before injury from decreased blood flow
occurs.

7. Answer: (A) Prevent stress ulcer
Rationale: Curlings ulcer occurs as a generalized stress response in burn patients. This
results in a decreased production of mucus and increased secretion of gastric acid. The best
treatment for this prophylactic use of antacids and H2 receptor blockers.

8. Answer: (D) Continue to monitor and record hourly urine output
Rationale: Normal urine output for an adult is approximately 1 ml/minute (60 ml/hour).
Therefore, this client's output is normal. Beyond continued evaluation, no nursing action is
warranted.

9. Answer: (B) My ankle feels warm.
Rationale: Ice application decreases pain and swelling. Continued or increased pain,
redness, and increased warmth are signs of inflammation that shouldn't occur after ice
application

10. Answer: (B) Hyperkalemia
Rationale: A loop diuretic removes water and, along with it, sodium and potassium. This
may result in hypokalemia, hypovolemia, and hyponatremia.

11. Answer:(A) Have condescending trust and confidence in their subordinates
Rationale: Benevolent-authoritative managers pretentiously show their trust and confidence
to their followers.

12. Answer: (A) Provides continuous, coordinated and comprehensive nursing services.
Rationale: Functional nursing is focused on tasks and activities and not on the care of the
patients.

13. Answer: (B) Standard written order
Rationale: This is a standard written order. Prescribers write a single order for medications
given only once. A stat order is written for
medications given immediately for an urgent client problem. A standing order, also known
as a protocol, establishes guidelines for treating a
particular disease or set of symptoms in special care areas such as the coronary care unit.
Facilities also may institute medication protocols that specifically designate drugs that a
nurse may not give.

14. Answer: (D) Liquid or semi-liquid stools
Rationale: Passage of liquid or semi-liquid stools results from seepage of unformed bowel
contents around the impacted stool in the rectum. Clients
with fecal impaction don't pass hard, brown, formed stools because the feces can't move
past the impaction. These clients typically report the urge
to defecate (although they can't pass stool) and a decreased appetite.

15. Answer: (C) Pulling the helix up and back
Rationale: To perform an otoscopic examination on an adult, the nurse grasps the helix of
the ear and pulls it up and back to straighten the ear canal. For a child, the nurse grasps
the helix and pulls it down to straighten the ear canal. Pulling the lobule in any direction
wouldn't straighten the ear canal for visualization.
16. Answer: (A) Protect the irritated skin from sunlight.
Rationale: Irradiated skin is very sensitive and must be protected with clothing or sunblock.
The priority approach is the avoidance of strong sunlight.
17. Answer: (C) Assist the client in removing dentures and nail polish.
Rationale: Dentures, hairpins, and combs must be removed. Nail polish must be removed so
that cyanosis can be easily monitored by observing the nail beds.
18. Answer: (D) Sudden onset of continuous epigastric and back pain.
Rationale: The autodigestion of tissue by the pancreatic enzymes results in pain from
inflammation, edema, and possible hemorrhage. Continuous, unrelieved epigastric or back
pain reflects the inflammatory process in the pancreas.
19. Answer: (B) Provide high-protein, high-carbohydrate diet.
Rationale: A positive nitrogen balance is important for meeting metabolic needs, tissue
repair, and resistance to infection. Caloric goals may be as high as 5000 calories per day.
20. Answer: (A) Blood pressure and pulse rate.
Rationale: The baseline must be established to recognize the signs of an anaphylactic or
hemolytic reaction to the transfusion.
21. Answer: (D) Immobilize the leg before moving the client.
Rationale: If the nurse suspects a fracture, splinting the area before moving the client is
imperative. The nurse should call for emergency help if the client is not hospitalized and call
for a physician for the hospitalized client.
22. Answer: (B) Admit the client into a private room.
Rationale: The client who has a radiation implant is placed in a private room and has a
limited number of visitors. This reduces the exposure of others to the radiation.
23. Answer: (C) Risk for infection
Rationale: Agranulocytosis is characterized by a reduced number of leukocytes (leucopenia)
and neutrophils (neutropenia) in the blood. The client is at high risk for infection because of
the decreased body defenses against microorganisms. Deficient knowledge related to the
nature of the disorder may be appropriate diagnosis but is not the priority.
24. Answer: (B) Place the client on the left side in the Trendelenburg position.
Rationale: Lying on the left side may prevent air from flowing into the pulmonary veins. The
Trendelenburg position increases intrathoracic pressure, which decreases the amount of
blood pulled into the vena cava during aspiration.
25. Answer: (A) Autocratic.
Rationale: The autocratic style of leadership is a task-oriented and directive.
26. Answer: (D) 2.5 cc
Rationale: 2.5 cc is to be added, because only a 500 cc bag of solution is being medicated
instead of a 1 liter.
27. Answer: (A) 50 cc/ hour
Rationale: A rate of 50 cc/hr. The child is to receive 400 cc over a period of 8 hours = 50
cc/hr.
28. Answer: (B) Assess the client for presence of pain.
Rationale: Assessing the client for pain is a very important measure. Postoperative pain is
an indication of complication. The nurse should also assess the client for pain to provide for
the clients comfort.
29. Answer: (A) BP 80/60, Pulse 110 irregular
Rationale: The classic signs of cardiogenic shock are low blood pressure, rapid and weak
irregular pulse, cold, clammy skin, decreased urinary output, and cerebral hypoxia.
30. Answer: (A) Take the proper equipment, place the client in a comfortable position, and
record the appropriate information in the clients chart.
Rationale: It is a general or comprehensive statement about the correct procedure, and it
includes the basic ideas which are found in the other options
31. Answer: (B) Evaluation
Rationale: Evaluation includes observing the person, asking questions, and comparing the
patients behavioral responses with the expected outcomes.
32. Answer: (C) History of present illness
Rationale: The history of present illness is the single most important factor in assisting the
health professional in arriving at a diagnosis or determining the persons needs.
33. Answer: (A) Trochanter roll extending from the crest of the ileum to the mid-thigh.
Rationale: A trochanter roll, properly placed, provides resistance to the external rotation of
the hip.
34. Answer: (C) Stage III
Rationale: Clinically, a deep crater or without undermining of adjacent tissue is noted.
35. Answer: (A) Second intention healing
Rationale: When wounds dehisce, they will allowed to heal by secondary intention
36. Answer: (D) Tachycardia
Rationale: With an extracellular fluid or plasma volume deficit, compensatory mechanisms
stimulate the heart, causing an increase in heart rate.
37. Answer: (A) 0.75
Rationale: To determine the number of milliliters the client should receive, the nurse uses
the fraction method in the following equation.
75 mg/X ml = 100 mg/1 ml
To solve for X, cross-multiply:
75 mg x 1 ml = X ml x 100 mg
75 = 100X
75/100 = X
0.75 ml (or ml) = X

38. Answer: (D) Its a measure of effect, not a standard measure of weight or quantity.
Rationale: An insulin unit is a measure of effect, not a standard measure of weight or
quantity. Different drugs measured in units may have no relationship to one another in
quality or quantity.
39. Answer: (B) 38.9 C
Rationale: To convert Fahrenheit degreed to Centigrade, use this formula
C = (F 32) 1.8
C = (102 32) 1.8
C = 70 1.8
C = 38.9
40. Answer: (C) Failing eyesight, especially close vision.
Rationale: Failing eyesight, especially close vision, is one of the first signs of aging in middle
life (ages 46 to 64). More frequent aches and pains begin in the early late years (ages 65 to
79). Increase in loss of muscle tone occurs in later years (age 80 and older).
41. Answer: (A) Checking and taping all connections
Rationale: Air leaks commonly occur if the system isnt secure. Checking all connections and
taping them will prevent air leaks. The chest drainage system is kept lower to promote
drainage not to prevent leaks.
42. Answer: (A) Check the clients identification band.
Rationale: Checking the clients identification band is the safest way to verify a clients
identity because the band is assigned on admission and isnt be removed at any time. (If it
is removed, it must be replaced). Asking the clients name or having the client repeated his
name would be appropriate only for a client whos alert, oriented, and able to understand
what is being said, but isnt the safe standard of practice. Names on bed arent always
reliable
43. Answer: (B) 32 drops/minute
Rationale: Giving 1,000 ml over 8 hours is the same as giving 125 ml over 1 hour (60
minutes). Find the number of milliliters per minute as follows:
125/60 minutes = X/1 minute
60X = 125 = 2.1 ml/minute
To find the number of drops per minute:
2.1 ml/X gtt = 1 ml/ 15 gtt
X = 32 gtt/minute, or 32 drops/minute

44. Answer: (A) Clamp the catheter
Rationale: If a central venous catheter becomes disconnected, the nurse should immediately
apply a catheter clamp, if available. If a clamp isnt available, the nurse can place a sterile
syringe or catheter plug in the catheter hub. After cleaning the hub with alcohol or
povidone-iodine solution, the nurse must replace the I.V. extension and restart the infusion.
45. Answer: (D) Auscultation, percussion, and palpation.
Rationale: The correct order of assessment for examining the abdomen is inspection,
auscultation, percussion, and palpation. The reason for this approach is that the less
intrusive techniques should be performed before the more intrusive techniques. Percussion
and palpation can alter natural findings during auscultation.
46. Answer: (D) Ulnar surface of the hand
Rationale: The nurse uses the ulnar surface, or ball, of the hand to asses tactile fremitus,
thrills, and vocal vibrations through the chest wall. The
fingertips and finger pads best distinguish texture and shape. The dorsal surface best feels
warmth.
47. Answer: (C) Formative
Rationale: Formative (or concurrent) evaluation occurs continuously throughout the
teaching and learning process. One benefit is that the nurse can adjust teaching strategies
as necessary to enhance learning. Summative, or retrospective, evaluation occurs at the
conclusion of the teaching and learning session. Informative is not a type of evaluation.

48. Answer: (B) Once per year
Rationale: Yearly mammograms should begin at age 40 and continue for
as long as the woman is in good health. If health risks, such as family
history, genetic tendency, or past breast cancer, exist, more frequent
examinations may be necessary.
49. Answer: (A) Respiratory acidosis
Rationale: The client has a below-normal (acidic) blood pH value and an above-normal
partial pressure of arterial carbon dioxide (Paco2) value, indicating respiratory acidosis. In
respiratory alkalosis, the pH value is above normal and in the Paco2 value is below normal.
In metabolic acidosis, the pH and bicarbonate (Hco3) values are below normal. In metabolic
alkalosis, the pH and Hco3 values are above normal.
50. Answer: (B) To provide support for the client and family in coping with terminal illness.
Rationale: Hospices provide supportive care for terminally ill clients and their families.
Hospice care doesnt focus on counseling regarding health care costs. Most client referred to
hospices have been treated for their disease without success and will receive only palliative
care in the hospice.
51. Answer: (C) Using normal saline solution to clean the ulcer and applying a protective
dressing as necessary.
Rationale: Washing the area with normal saline solution and applying a protective dressing
are within the nurses realm of interventions and will protect the area. Using a povidone-
iodine wash and an antibiotic cream require a physicians order. Massaging with an
astringent can further damage the skin.
52. Answer: (D) Foot
Rationale: An elastic bandage should be applied form the distal area to the proximal area.
This method promotes venous return. In this case, the nurse should begin applying the
bandage at the clients foot. Beginning at the ankle, lower thigh, or knee does not promote
venous return.
53. Answer: (B) Hypokalemia
Rationale: Insulin administration causes glucose and potassium to move into the cells,
causing hypokalemia.
54. Answer: (A) Throbbing headache or dizziness
Rationale: Headache and dizziness often occur when nitroglycerin is taken at the beginning
of therapy. However, the client usually develops tolerance

55. Answer: (D) Check the clients level of consciousness
Rationale: Determining unresponsiveness is the first step assessment action to take. When
a client is in ventricular tachycardia, there is a significant decrease in cardiac output.
However, checking the unresponsiveness ensures whether the client is affected by the
decreased cardiac output.
56. Answer: (B) On the affected side of the client.
Rationale: When walking with clients, the nurse should stand on the affected side and grasp
the security belt in the midspine area of the small of the back. The nurse should position the
free hand at the shoulder area so that the client can be pulled toward the nurse in the event
that there is a forward fall. The client is instructed to look up and outward rather than at his
or her feet.
57. Answer: (A) Urine output: 45 ml/hr
Rationale: Adequate perfusion must be maintained to all vital organs in order for the client
to remain visible as an organ donor. A urine output of 45 ml per hour indicates adequate
renal perfusion. Low blood pressure and delayed capillary refill time are circulatory system
indicators of inadequate perfusion. A serum pH of 7.32 is acidotic, which adversely affects
all body tissues.
58. Answer: (D ) Obtaining the specimen from the urinary drainage bag.
Rationale: A urine specimen is not taken from the urinary drainage bag. Urine undergoes
chemical changes while sitting in the bag and does not necessarily reflect the current client
status. In addition, it may become contaminated with bacteria from opening the system.
59. Answer: (B) Cover the client, place the call light within reach, and answer the phone
call.
Rationale: Because telephone call is an emergency, the nurse may need to answer it. The
other appropriate action is to ask another nurse to accept the call. However, is not one of
the options. To maintain privacy and safety, the nurse covers the client and places the call
light within the clients reach. Additionally, the clients door should be closed or the room
curtains pulled around the bathing area.
60. Answer: (C) Use a sterile plastic container for obtaining the specimen.
Rationale: Sputum specimens for culture and sensitivity testing need to be obtained using
sterile techniques because the test is done to determine the presence of organisms. If the
procedure for obtaining the specimen is not sterile, then the specimen is not sterile, then
the specimen would be contaminated and the results of the test would be invalid.

61. Answer: (A) Puts all the four points of the walker flat on the floor, puts weight on the
hand pieces, and then walks into it.
Rationale: When the client uses a walker, the nurse stands adjacent to the affected side.
The client is instructed to put all four points of the walker 2 feet forward flat on the floor
before putting weight on hand pieces. This will ensure client safety and prevent stress
cracks in the walker. The client is then instructed to move the walker forward and walk into
it.
62. Answer: (C) Draws one line to cross out the incorrect information and then initials the
change.
Rationale: To correct an error documented in a medical record, the nurse draws one line
through the incorrect information and then initials the error. An error is never erased and
correction fluid is never used in the medical record.
63. Answer: (C) Secures the client safety belts after transferring to the stretcher.
Rationale: During the transfer of the client after the surgical procedure is complete, the
nurse should avoid exposure of the client because of the risk for potential heat loss. Hurried
movements and rapid changes in the position should be avoided because these predispose
the client to hypotension. At the time of the transfer from the surgery table to the stretcher,
the client is still affected by the effects of the anesthesia; therefore, the client should not
move self. Safety belts can prevent the client from falling off the stretcher.
64. Answer: (B) Gown and gloves
Rationale: Contact precautions require the use of gloves and a gown if direct client contact
is anticipated. Goggles are not necessary unless the
nurse anticipates the splashes of blood, body fluids, secretions, or excretions may occur.
Shoe protectors are not necessary.
65. Answer: (C) Quad cane
Rationale: Crutches and a walker can be difficult to maneuver for a client with weakness on
one side. A cane is better suited for client with weakness of the arm and leg on one side.
However, the quad cane would provide the most stability because of the structure of the
cane and because a quad cane has four legs.
66. Answer: (D) Left side-lying with the head of the bed elevated 45 degrees.
Rationale: To facilitate removal of fluid from the chest wall, the client is positioned sitting at
the edge of the bed leaning over the bedside table with the feet supported on a stool. If the
client is unable to sit up, the client is positioned lying in bed on the unaffected side with the
head of the bed elevated 30 to 45 degrees.

67. Answer: (D) Reliability
Rationale: Reliability is consistency of the research instrument. It refers to
the repeatability of the instrument in extracting the same responses upon
its repeated administration.
68. Answer: (A) Keep the identities of the subject secret
Rationale: Keeping the identities of the research subject secret will ensure anonymity
because this will hinder providing link between the information given to whoever is its
source.
69. Answer: (A) Descriptive- correlational
Rationale: Descriptive- correlational study is the most appropriate for this study because it
studies the variables that could be the antecedents of the increased incidence of nosocomial
infection.
70. Answer: (C) Use of laboratory data
Rationale: Incidence of nosocomial infection is best collected through the use of
biophysiologic measures, particularly in vitro measurements, hence laboratory data is
essential.
71. Answer: (B) Quasi-experiment
Rationale: Quasi-experiment is done when randomization and control of the variables are
not possible.
72. Answer: (C) Primary source
Rationale: This refers to a primary source which is a direct account of the investigation done
by the investigator. In contrast to this is a secondary source, which is written by someone
other than the original researcher.
73. Answer: (A) Non-maleficence
Rationale: Non-maleficence means do not cause harm or do any action that will cause any
harm to the patient/client. To do good is referred as beneficence.
74. Answer: (C) Res ipsa loquitor
Rationale: Res ipsa loquitor literally means the thing speaks for itself. This means in
operational terms that the injury caused is the proof that there was a negligent act.
75. Answer: (B) The Board can investigate violations of the nursing law and code of ethics
Rationale: Quasi-judicial power means that the Board of Nursing has the authority to
investigate violations of the nursing law and can issue summons, subpoena or subpoena
duces tecum as needed.

76. Answer: (C) May apply for re-issuance of his/her license based on certain conditions
stipulated in RA 9173
Rationale: RA 9173 sec. 24 states that for equity and justice, a revoked license maybe re-
issued provided that the following conditions are met: a)
the cause for revocation of license has already been corrected or removed; and, b) at least
four years has elapsed since the license has been revoked.
77. Answer: (B) Review related literature
Rationale: After formulating and delimiting the research problem, the researcher conducts a
review of related literature to determine the extent of what has been done on the study by
previous researchers.
78. Answer: (B) Hawthorne effect
Rationale: Hawthorne effect is based on the study of Elton Mayo and company about the
effect of an intervention done to improve the working conditions of the workers on their
productivity. It resulted to an increased productivity but not due to the intervention but due
to the psychological effects of being observed. They performed differently because they
were under observation.
79. Answer: (B) Determines the different nationality of patients frequently admitted and
decides to get representations samples from each.
Rationale: Judgment sampling involves including samples according to the knowledge of the
investigator about the participants in the study.
80. Answer: (B) Madeleine Leininger
Rationale: Madeleine Leininger developed the theory on transcultural theory based on her
observations on the behavior of selected people within a culture.
81. Answer: (A) Random
Rationale: Random sampling gives equal chance for all the elements in the population to be
picked as part of the sample.
82. Answer: (A) Degree of agreement and disagreement
Rationale: Likert scale is a 5-point summated scale used to determine the degree of
agreement or disagreement of the respondents to a statement in a study
83. Answer: (B) Sr. Callista Roy
Rationale: Sr. Callista Roy developed the Adaptation Model which involves the physiologic
mode, self-concept mode, role function mode and dependence mode.
84. Answer: (A) Span of control
Rationale: Span of control refers to the number of workers who report directly to a
manager.
85. Answer: (B) Autonomy
Rationale: Informed consent means that the patient fully understands about the surgery,
including the risks involved and the alternative solutions. In giving consent it is done with
full knowledge and is given freely. The action of allowing the patient to decide whether a
surgery is to be done or not exemplifies the bioethical principle of autonomy.
86. Answer: (C) Avoid wearing canvas shoes.
Rationale: The client should be instructed to avoid wearing canvas shoes. Canvas shoes
cause the feet to perspire, which may, in turn, cause skin
irritation and breakdown. Both cotton and cornstarch absorb perspiration. The client should
be instructed to cut toenails straight across with nail
clippers.

87. Answer: (D) Ground beef patties
Rationale: Meat is an excellent source of complete protein, which this client needs to repair
the tissue breakdown caused by pressure ulcers.
Oranges and broccoli supply vitamin C but not protein. Ice cream supplies only some
incomplete protein, making it less helpful in tissue repair.

88. Answer: (D) Sims left lateral
Rationale: The Sims' left lateral position is the most common position used to administer a
cleansing enema because it allows gravity to aid the flow of fluid along the curve of the
sigmoid colon. If the client can't assume this position nor has poor sphincter control, the
dorsal recumbent or right lateral position may be used. The supine and prone positions are
inappropriate and uncomfortable for the client.

89. Answer: (A) Arrange for typing and cross matching of the clients blood.
Rationale: The nurse first arranges for typing and cross matching of the client's blood to
ensure compatibility with donor blood. The other options,
although appropriate when preparing to administer a blood transfusion, come later.

90. Answer: (A) Independent
Rationale: Nursing interventions are classified as independent, interdependent, or
dependent. Altering the drug schedule to coincide with the client's daily routine represents
an independent intervention, whereas consulting with the physician and pharmacist to
change a client's medication because of adverse reactions represents an interdependent
intervention. Administering an already-prescribed drug on time is a dependent intervention.
An intradependent nursing intervention doesn't exist.

91. Answer: (D) Evaluation
Rationale: The nursing actions described constitute evaluation of the expected outcomes.
The findings show that the expected outcomes have been achieved. Assessment consists of
the client's history, physical examination, and laboratory studies. Analysis consists of
considering assessment information to derive the appropriate nursing diagnosis.
Implementation is the phase of the nursing process where the nurse puts the plan of care
into action.

92. Answer: (B) To observe the lower extremities
Rationale: Elastic stockings are used to promote venous return. The nurse needs to remove
them once per day to observe the condition of the skin underneath the stockings. Applying
the stockings increases blood flow to the heart. When the stockings are in place, the leg
muscles can still stretch and relax, and the veins can fill with blood.

93. Answer:(A) Instructing the client to report any itching, swelling, or dyspnea.
Rationale: Because administration of blood or blood products may cause serious adverse
effects such as allergic reactions, the nurse must monitor the client for these effects. Signs
and symptoms of life-threatening allergic reactions include itching, swelling, and dyspnea.
Although the nurse should inform the client of the duration of the transfusion and should
document its administration, these actions are less critical to the client's immediate health.
The nurse should assess vital signs at least hourly during the transfusion.

94. Answer: (B) Decrease the rate of feedings and the concentration of the formula.
Rationale: Complaints of abdominal discomfort and nausea are common in clients receiving
tube feedings. Decreasing the rate of the feeding and the concentration of the formula
should decrease the client's discomfort. Feedings are normally given at room temperature to
minimize abdominal cramping. To prevent aspiration during feeding, the head of the client's
bed should be elevated at least 30 degrees. Also, to prevent bacterial growth, feeding
containers should be routinely changed every 8 to 12 hours.

95. Answer: (D) Roll the vial gently between the palms.
Rationale: Rolling the vial gently between the palms produces heat, which helps dissolve the
medication. Doing nothing or inverting the vial wouldn't help dissolve the medication.
Shaking the vial vigorously could cause the medication to break down, altering its action.

96. Answer: (B) Assist the client to the semi-Fowler position if possible.
Rationale: By assisting the client to the semi-Fowler position, the nurse promotes easier
chest expansion, breathing, and oxygen intake. The nurse should secure the elastic band so
that the face mask fits comfortably and snugly rather than tightly, which could lead to
irritation. The nurse should apply the face mask from the client's nose down to the chin
not vice versa. The nurse should check the connectors between the oxygen equipment and
humidifier to ensure that they're airtight; loosened connectors can cause loss of oxygen.

97. Answer: (B) 4 hours
Rationale: A unit of packed RBCs may be given over a period of between 1 and 4 hours. It
shouldn't infuse for longer than 4 hours because the risk of contamination and sepsis
increases after that time. Discard or return to the blood bank any blood not given within this
time, according to facility policy.

98. Answer: (B) Immediately before administering the next dose.
Rationale: Measuring the blood drug concentration helps determine whether the dosing has
achieved the therapeutic goal. For measurement of the trough, or lowest, blood level of a
drug, the nurse draws a blood sample immediately before administering the next dose.
Depending on the drug's duration of action and half-life, peak blood drug levels typically are
drawn after administering the next dose.

99. Answer: (A) The nurse can implement medication orders quickly.
Rationale: A floor stock system enables the nurse to implement medication orders quickly.
It doesn't allow for pharmacist input, nor does it minimize transcription errors or reinforce
accurate calculations.

100. Answer: (C) Shifting dullness over the abdomen.
Rationale: Shifting dullness over the abdomen indicates ascites, an abnormal finding. The
other options are normal abdominal findings.










1. Which element in the circular chain of infection can be eliminated by preserving skin
integrity?

a. Host
b. Reservoir
c. Mode of transmission
d. Portal of entry


2. Which of the following will probably result in a break in sterile technique for respiratory
isolation?

a. Opening the patients window to the outside environment
b. Turning on the patients room ventilator
c. Opening the door of the patients room leading into the hospital corridor
d. Failing to wear gloves when administering a bed bath


3. Which of the following patients is at greater risk for contracting an infection?

a. A patient with leukopenia
b. A patient receiving broad-spectrum antibiotics
c. A postoperative patient who has undergone orthopedic surgery
d. A newly diagnosed diabetic patient


4. Effective hand washing requires the use of:

a. Soap or detergent to promote emulsification
b. Hot water to destroy bacteria
c. A disinfectant to increase surface tension
d. All of the above


5. After routine patient contact, hand washing should last at least:

a. 30 seconds
b. 1 minute
c. 2 minute
d. 3 minutes


6. Which of the following procedures always requires surgical asepsis?

a. Vaginal instillation of conjugated estrogen
b. Urinary catheterization
c. Nasogastric tube insertion
d. Colostomy irrigation


7. Sterile technique is used whenever:

a. Strict isolation is required
b. Terminal disinfection is performed
c. Invasive procedures are performed
d. Protective isolation is necessary


8. Which of the following constitutes a break in sterile technique while preparing a sterile
field for a dressing change?

a. Using sterile forceps, rather than sterile gloves, to handle a sterile item
b. Touching the outside wrapper of sterilized material without sterile gloves
c. Placing a sterile object on the edge of the sterile field
d. Pouring out a small amount of solution (15 to 30 ml) before pouring the solution into a
sterile container


9. A natural body defense that plays an active role in preventing infection is:

a. Yawning
b. Body hair
c. Hiccupping
d. Rapid eye movements


10. All of the following statement are true about donning sterile gloves except:

a. The first glove should be picked up by grasping the inside of the cuff.
b. The second glove should be picked up by inserting the gloved fingers under the cuff
outside the glove.
c. The gloves should be adjusted by sliding the gloved fingers under the sterile cuff and
pulling the glove over the wrist
d. The inside of the glove is considered sterile


11.When removing a contaminated gown, the nurse should be careful that the first thing
she touches is the:

a. Waist tie and neck tie at the back of the gown
b. Waist tie in front of the gown
c. Cuffs of the gown
d. Inside of the gown


12.Which of the following nursing interventions is considered the most effective form or
universal precautions?

a. Cap all used needles before removing them from their syringes
b. Discard all used uncapped needles and syringes in an impenetrable protective container
c. Wear gloves when administering IM injections
d. Follow enteric precautions


13.All of the following measures are recommended to prevent pressure ulcers except:

a. Massaging the reddened are with lotion
b. Using a water or air mattress
c. Adhering to a schedule for positioning and turning
d. Providing meticulous skin care


14.Which of the following blood tests should be performed before a blood transfusion?

a. Prothrombin and coagulation time
b. Blood typing and cross-matching
c. Bleeding and clotting time
d. Complete blood count (CBC) and electrolyte levels.


15.The primary purpose of a platelet count is to evaluate the:

a. Potential for clot formation
b. Potential for bleeding
c. Presence of an antigen-antibody response
d. Presence of cardiac enzymes


16.Which of the following white blood cell (WBC) counts clearly indicates leukocytosis?

a. 4,500/mm
b. 7,000/mm
c. 10,000/mm
d. 25,000/mm


17. After 5 days of diuretic therapy with 20mg of furosemide (Lasix) daily, a patient begins
to exhibit fatigue, muscle cramping and muscle weakness. These symptoms probably
indicate that the patient is experiencing:

a. Hypokalemia
b. Hyperkalemia
c. Anorexia
d. Dysphagia


18.Which of the following statements about chest X-ray is false?

a. No contradictions exist for this test
b. Before the procedure, the patient should remove all jewelry, metallic objects, and buttons
above the waist
c. A signed consent is not required
d. Eating, drinking, and medications are allowed before this test


19.The most appropriate time for the nurse to obtain a sputum specimen for culture is:

a. Early in the morning
b. After the patient eats a light breakfast
c. After aerosol therapy
d. After chest physiotherapy


20.A patient with no known allergies is to receive penicillin every 6 hours. When
administering the medication, the nurse observes a fine rash on the
patients skin. The most appropriate nursing action would be to:

a. Withhold the moderation and notify the physician
b. Administer the medication and notify the physician
c. Administer the medication with an antihistamine
d. Apply corn starch soaks to the rash


21.All of the following nursing interventions are correct when using the Ztrack method of
drug injection except:

a. Prepare the injection site with alcohol
b. Use a needle thats a least 1 long
c. Aspirate for blood before injection
d. Rub the site vigorously after the injection to promote absorption


22.The correct method for determining the vastus lateralis site for I.M. injection is to:

a. Locate the upper aspect of the upper outer quadrant of the buttock about 5 to 8 cm
below the iliac crest
b. Palpate the lower edge of the acromion process and the midpoint lateral aspect of the
arm
c. Palpate a 1 circular area anterior to the umbilicus
d. Divide the area between the greater femoral trochanter and the lateral femoral condyle
into thirds, and select the middle third on the anterior of the thigh


23.The mid-deltoid injection site is seldom used for I.M. injections because it:

a. Can accommodate only 1 ml or less of medication
b. Bruises too easily
c. Can be used only when the patient is lying down
d. Does not readily parenteral medication


24.The appropriate needle size for insulin injection is:

a. 18G, 1 long
b. 22G, 1 long
c. 22G, 1 long
d. 25G, 5/8 long


25.The appropriate needle gauge for intradermal injection is:

a. 20G
b. 22G
c. 25G
d. 26G


26.Parenteral penicillin can be administered as an:

a. IM injection or an IV solution
b. IV or an intradermal injection
c. Intradermal or subcutaneous injection
d. IM or a subcutaneous injection


27.The physician orders gr 10 of aspirin for a patient. The equivalent dose in milligrams is:

a. 0.6 mg
b. 10 mg
c. 60 mg
d. 600 mg


28.The physician orders an IV solution of dextrose 5% in water at 100ml/hour. What would
the flow rate be if the drop factor is 15 gtt = 1 ml?

a. 5 gtt/minute
b. 13 gtt/minute
c. 25 gtt/minute
d. 50 gtt/minute


29.Which of the following is a sign or symptom of a hemolytic reaction to blood transfusion?

a. Hemoglobinuria
b. Chest pain
c. Urticaria
d. Distended neck veins


30.Which of the following conditions may require fluid restriction?

a. Fever
b. Chronic Obstructive Pulmonary Disease
c. Renal Failure
d. Dehydration


31.All of the following are common signs and symptoms of phlebitis except:

a. Pain or discomfort at the IV insertion site
b. Edema and warmth at the IV insertion site
c. A red streak exiting the IV insertion site
d. Frank bleeding at the insertion site


32.The best way of determining whether a patient has learned to instill ear medication
properly is for the nurse to:

a. Ask the patient if he/she has used ear drops before
b. Have the patient repeat the nurses instructions using her own words
c. Demonstrate the procedure to the patient and encourage to ask questions
d. Ask the patient to demonstrate the procedure


33.Which of the following types of medications can be administered via gastrostomy tube?

a. Any oral medications
b. Capsules whole contents are dissolve in water
c. Enteric-coated tablets that are thoroughly dissolved in water
d. Most tablets designed for oral use, except for extended-duration compounds


34.A patient who develops hives after receiving an antibiotic is exhibiting drug:

a. Tolerance
b. Idiosyncrasy
c. Synergism
d. Allergy


35.A patient has returned to his room after femoral arteriography. All of the following are
appropriate nursing interventions except:

a. Assess femoral, popliteal, and pedal pulses every 15 minutes for 2 hours
b. Check the pressure dressing for sanguineous drainage
c. Assess a vital signs every 15 minutes for 2 hours
d. Order a hemoglobin and hematocrit count 1 hour after the arteriography


36.The nurse explains to a patient that a cough:

a. Is a protective response to clear the respiratory tract of irritants
b. Is primarily a voluntary action
c. Is induced by the administration of an antitussive drug
d. Can be inhibited by splinting the abdomen


37.An infected patient has chills and begins shivering. The best nursing intervention is to:

a. Apply iced alcohol sponges
b. Provide increased cool liquids
c. Provide additional bedclothes
d. Provide increased ventilation


38.A clinical nurse specialist is a nurse who has:

a. Been certified by the National League for Nursing
b. Received credentials from the Philippine Nurses Association
c. Graduated from an associate degree program and is a registered professional nurse
d. Completed a masters degree in the prescribed clinical area and is a registered
professional nurse.


39.The purpose of increasing urine acidity through dietary means is to:

a. Decrease burning sensations
b. Change the urines color
c. Change the urines concentration
d. Inhibit the growth of microorganisms


40.Clay colored stools indicate:

a. Upper GI bleeding
b. Impending constipation
c. An effect of medication
d. Bile obstruction


41.In which step of the nursing process would the nurse ask a patient if the medication she
administered relieved his pain?

a. Assessment
b. Analysis
c. Planning
d. Evaluation


42.All of the following are good sources of vitamin A except:

a. White potatoes
b. Carrots
c. Apricots
d. Egg yolks


43.Which of the following is a primary nursing intervention necessary for all patients with a
Foley Catheter in place?

a. Maintain the drainage tubing and collection bag level with the patients bladder
b. Irrigate the patient with 1% Neosporin solution three times a daily
c. Clamp the catheter for 1 hour every 4 hours to maintain the bladders elasticity
d. Maintain the drainage tubing and collection bag below bladder level to facilitate drainage
by gravity


44.The ELISA test is used to:

a. Screen blood donors for antibodies to human immunodeficiency virus (HIV)
b. Test blood to be used for transfusion for HIV antibodies
c. Aid in diagnosing a patient with AIDS
d. All of the above


45.The two blood vessels most commonly used for TPN infusion are the:

a. Subclavian and jugular veins
b. Brachial and subclavian veins
c. Femoral and subclavian veins
d. Brachial and femoral veins


46.Effective skin disinfection before a surgical procedure includes which of the following
methods?

a. Shaving the site on the day before surgery
b. Applying a topical antiseptic to the skin on the evening before surgery
c. Having the patient take a tub bath on the morning of surgery
d. Having the patient shower with an antiseptic soap on the evening v=before and the
morning of surgery


47.When transferring a patient from a bed to a chair, the nurse should use which muscles to
avoid back injury?

a. Abdominal muscles
b. Back muscles
c. Leg muscles
d. Upper arm muscles


48.Thrombophlebitis typically develops in patients with which of the following conditions?

a. Increases partial thromboplastin time
b. Acute pulsus paradoxus
c. An impaired or traumatized blood vessel wall
d. Chronic Obstructive Pulmonary Disease (COPD)


49.In a recumbent, immobilized patient, lung ventilation can become altered, leading to
such respiratory complications as:

a. Respiratory acidosis, ateclectasis, and hypostatic pneumonia
b. Appneustic breathing, atypical pneumonia and respiratory alkalosis
c. Cheyne-Strokes respirations and spontaneous pneumothorax
d. Kussmails respirations and hypoventilation


50.Immobility impairs bladder elimination, resulting in such disorders as

a. Increased urine acidity and relaxation of the perineal muscles, causing incontinence
b. Urine retention, bladder distention, and infection
c. Diuresis, natriuresis, and decreased urine specific gravity
d. Decreased calcium and phosphate levels in the urine

















1. D. In the circular chain of infection, pathogens must be able to leave their reservoir and
be transmitted to a susceptible host through a portal of entry, such as broken skin.

2. C. Respiratory isolation, like strict isolation, requires that the door to the door patients
room remain closed. However, the patients room should be well ventilated, so opening the
window or turning on the ventricular is desirable. The nurse does not need to wear gloves
for respiratory isolation, but good hand washing is important for all types of isolation.

3. A. Leukopenia is a decreased number of leukocytes (white blood cells), which are
important in resisting infection. None of the other situations would put the patient at risk for
contracting an infection; taking broadspectrum antibiotics might actually reduce the
infection risk.

4. A. Soaps and detergents are used to help remove bacteria because of their ability to
lower the surface tension of water and act as emulsifying agents. Hot water may lead to
skin irritation or burns.

5. A. Depending on the degree of exposure to pathogens, hand washing may last from 10
seconds to 4 minutes. After routine patient contact, hand washing for 30 seconds effectively
minimizes the risk of pathogen transmission.

6. B. The urinary system is normally free of microorganisms except at the urinary meatus.
Any procedure that involves entering this system must use surgically aseptic measures to
maintain a bacteria-free state.

7. C. All invasive procedures, including surgery, catheter insertion, and administration of
parenteral therapy, require sterile technique to maintain a sterile environment. All
equipment must be sterile, and the nurse and the physician must wear sterile gloves and
maintain surgical asepsis. In the operating room, the nurse and physician are required to
wear sterile gowns, gloves, masks, hair covers, and shoe covers for all invasive procedures.
Strict isolation requires the use of clean gloves, masks, gowns and equipment to prevent
the transmission of highly communicable diseases by contact or by airborne routes.
Terminal disinfection is the disinfection of all contaminated supplies and equipment after a
patient has been discharged to prepare them for reuse by another patient. The purpose of
protective (reverse) isolation is to prevent a person with seriously impaired resistance from
coming into contact who potentially pathogenic organisms.

8. C. The edges of a sterile field are considered contaminated. When sterile items are
allowed to come in contact with the edges of the field, the sterile items also become
contaminated.

9. B. Hair on or within body areas, such as the nose, traps and holds particles that contain
microorganisms. Yawning and hiccupping do not prevent microorganisms from entering or
leaving the body. Rapid eye movement marks the stage of sleep during which dreaming
occurs.

10. D. The inside of the glove is always considered to be clean, but not sterile.

11. A. The back of the gown is considered clean, the front is contaminated. So, after
removing gloves and washing hands, the nurse should untie the back of the gown; slowly
move backward away from the gown, holding the inside of the gown and keeping the edges
off the floor; turn and fold the gown inside out; discard it in a contaminated linen container;
then wash her hands again.

12. B. According to the Centers for Disease Control (CDC), blood-to-blood contact occurs
most commonly when a health care worker attempts to cap a used needle. Therefore, used
needles should never be recapped; instead they should be inserted in a specially designed
puncture resistant, labeled container. Wearing gloves is not always necessary
when administering an I.M. injection. Enteric precautions prevent the transfer of pathogens
via feces.

13. A. Nurses and other health care professionals previously believed that massaging a
reddened area with lotion would promote venous return and reduce edema to the area.
However, research has shown that massage only increases the likelihood of cellular
ischemia and necrosis to the area.

14. B. Before a blood transfusion is performed, the blood of the donor and recipient must be
checked for compatibility. This is done by blood typing (a test that determines a persons
blood type) and cross-matching (a procedure that determines the compatibility of the
donors and recipients blood after the blood types has been matched). If the blood
specimens are incompatible, hemolysis and antigen-antibody reactions will occur.

15. A. Platelets are disk-shaped cells that are essential for blood coagulation. A platelet
count determines the number of thrombocytes in blood available for promoting hemostasis
and assisting with blood coagulation after injury. It also is used to evaluate the patients
potential for bleeding; however, this is not its primary purpose. The normal count ranges
from 150,000 to 350,000/mm3. A count of 100,000/mm3 or less indicates a potential
for bleeding; count of less than 20,000/mm3 is associated with spontaneous bleeding.

16. D. Leukocytosis is any transient increase in the number of white blood cells (leukocytes)
in the blood. Normal WBC counts range from 5,000 to 100,000/mm3. Thus, a count of
25,000/mm3 indicates leukocytosis.

17. A. Fatigue, muscle cramping, and muscle weaknesses are symptoms of hypokalemia (an
inadequate potassium level), which is a potential side effect of diuretic therapy. The
physician usually orders supplemental potassium to prevent hypokalemia in patients
receiving diuretics. Anorexia is another symptom of hypokalemia. Dysphagia means
difficulty
swallowing.

18. A. Pregnancy or suspected pregnancy is the only contraindication for a chest X-ray.
However, if a chest X-ray is necessary, the patient can wear a lead apron to protect the
pelvic region from radiation. Jewelry, metallic objects, and buttons would interfere with the
X-ray and thus should not be worn above the waist. A signed consent is not required
because a chest X-ray is not an invasive examination. Eating, drinking and medications
are allowed because the X-ray is of the chest, not the abdominal region.

19. A. Obtaining a sputum specimen early in this morning ensures an adequate supply of
bacteria for culturing and decreases the risk of contamination from food or medication.

20. A. Initial sensitivity to penicillin is commonly manifested by a skin rash, even in
individuals who have not been allergic to it previously. Because of the danger of
anaphylactic shock, he nurse should withhold the drug and
notify the physician, who may choose to substitute another drug. Administering an
antihistamine is a dependent nursing intervention that requires a written physicians order.
Although applying corn starch to the rash may relieve discomfort, it is not the nurses top
priority in such a potentially life-threatening situation.

21. D. The Z-track method is an I.M. injection technique in which the patients skin is pulled
in such a way that the needle track is sealed off after the injection. This procedure seals
medication deep into the muscle, thereby minimizing skin staining and irritation. Rubbing
the injection site is contraindicated because it may cause the medication to extravasate
into the skin.

22. D. The vastus lateralis, a long, thick muscle that extends the full length of the thigh, is
viewed by many clinicians as the site of choice for I.M. injections because it has relatively
few major nerves and blood vessels. The middle third of the muscle is recommended as the
injection site. The patient can be in a supine or sitting position for an injection into this site.

23. A. The mid-deltoid injection site can accommodate only 1 ml or less of medication
because of its size and location (on the deltoid muscle of the arm, close to the brachial
artery and radial nerve).

24. D. A 25G, 5/8 needle is the recommended size for insulin injection because insulin is
administered by the subcutaneous route. An 18G, 1 needle is usually used for I.M.
injections in children, typically in the vastus lateralis. A 22G, 1 needle is usually used for
adult I.M. injections, which are typically administered in the vastus lateralis or ventrogluteal
site.

25. D. Because an intradermal injection does not penetrate deeply into the skin, a small-
bore 25G needle is recommended. This type of injection is used primarily to administer
antigens to evaluate reactions for allergy or
sensitivity studies. A 20G needle is usually used for I.M. injections of oilbased medications;
a 22G needle for I.M. injections; and a 25G needle, for I.M. injections; and a 25G needle,
for subcutaneous insulin injections.

26. A. Parenteral penicillin can be administered I.M. or added to a solution and given I.V. It
cannot be administered subcutaneously or intradermally.

27. D. gr 10 x 60mg/gr 1 = 600 mg

28. C. 100ml/60 min X 15 gtt/ 1 ml = 25 gtt/minute

29. A. Hemoglobinuria, the abnormal presence of hemoglobin in the urine, indicates a
hemolytic reaction (incompatibility of the donors and recipients blood). In this reaction,
antibodies in the recipients plasma combine rapidly with donor RBCs; the cells are
hemolyzed in either circulatory or reticuloendothelial system. Hemolysis occurs more rapidly
in
ABO incompatibilities than in Rh incompatibilities. Chest pain and urticaria may be
symptoms of impending anaphylaxis. Distended neck veins are an indication of
hypervolemia.

30. C. In real failure, the kidney loses their ability to effectively eliminate wastes and fluids.
Because of this, limiting the patients intake of oral and I.V. fluids may be necessary. Fever,
chronic obstructive pulmonary disease, and dehydration are conditions for which fluids
should be encouraged.

31. D. Phlebitis, the inflammation of a vein, can be caused by chemical irritants (I.V.
solutions or medications), mechanical irritants (the needle or catheter used during
venipuncture or cannulation), or a localized allergic
reaction to the needle or catheter. Signs and symptoms of phlebitis include pain or
discomfort, edema and heat at the I.V. insertion site, and a red streak going up the arm or
leg from the I.V. insertion site.

32. D. Return demonstration provides the most certain evidence for evaluating the
effectiveness of patient teaching.

33. D. Capsules, enteric-coated tablets, and most extended duration or sustained release
products should not be dissolved for use in a gastrostomy tube. They are pharmaceutically
manufactured in these forms for valid reasons, and altering them destroys their purpose.
The nurse should seek an alternate physicians order when an ordered medication
is inappropriate for delivery by tube.

34. D. A drug-allergy is an adverse reaction resulting from an immunologic response
following a previous sensitizing exposure to the drug. The reaction can range from a rash or
hives to anaphylactic shock. Tolerance to a drug means that the patient experiences a
decreasing physiologic response to repeated administration of the drug in the same
dosage. Idiosyncrasy is an individuals unique hypersensitivity to a drug, food, or other
substance; it appears to be genetically determined. Synergism, is a drug interaction in
which the sum of the drugs combined effects is greater
than that of their separate effects.

35. D. A hemoglobin and hematocrit count would be ordered by the physician if bleeding
were suspected. The other answers are appropriate nursing interventions for a patient who
has undergone femoral arteriography.

36. A. Coughing, a protective response that clears the respiratory tract of irritants, usually
is involuntary; however it can be voluntary, as when a patient is taught to perform coughing
exercises. An antitussive drug inhibits coughing. Splinting the abdomen supports the
abdominal muscles when a patient coughs.

37. C. In an infected patient, shivering results from the bodys attempt to increase heat
production and the production of neutrophils and phagocytotic action through increased
skeletal muscle tension and contractions. Initial vasoconstriction may cause skin to feel cold
to the touch. Applying additional bed clothes helps to equalize the body
temperature and stop the chills. Attempts to cool the body result in further shivering,
increased metabloism, and thus increased heat production.

38. D. A clinical nurse specialist must have completed a masters degree in a clinical
specialty and be a registered professional nurse. The National League of Nursing accredits
educational programs in nursing and provides a testing service to evaluate student nursing
competence but it does not certify nurses. The American Nurses Association identifies
requirements for certification and offers examinations for certification in many areas
of nursing., such as medical surgical nursing. These certification (credentialing)
demonstrates that the nurse has the knowledge and the
ability to provide high quality nursing care in the area of her certification. A graduate of an
associate degree program is not a clinical nurse specialist: however, she is prepared to
provide bed side nursing with a high degree of knowledge and skill. She must successfully
complete the licensing examination to become a registered professional nurse.

39. D. Microorganisms usually do not grow in an acidic environment.

40. D. Bile colors the stool brown. Any inflammation or obstruction that impairs bile flow will
affect the stool pigment, yielding light, clay-colored stool. Upper GI bleeding results in black
or tarry stool. Constipation is characterized by small, hard masses. Many medications and
foods will discolor stool for example, drugs containing iron turn stool black.; beets
turn stool red.

41. D. In the evaluation step of the nursing process, the nurse must decide whether the
patient has achieved the expected outcome that was identified in the planning phase.

42. A. The main sources of vitamin A are yellow and green vegetables (such as carrots,
sweet potatoes, squash, spinach, collard greens, broccoli, and cabbage) and yellow fruits
(such as apricots, and cantaloupe). Animal sources include liver, kidneys, cream, butter,
and egg yolks.

43. D. Maintaing the drainage tubing and collection bag level with the patients bladder
could result in reflux of urine into the kidney. Irrigating the bladder with Neosporin and
clamping the catheter for 1 hour every 4 hours must
be prescribed by a physician.

44. D. The ELISA test of venous blood is used to assess blood and potential blood donors to
human immunodeficiency virus (HIV). A positive ELISA test combined with various signs
and symptoms helps to diagnose acquired immunodeficiency syndrome (AIDS)

45. D. Tachypnea (an abnormally rapid rate of breathing) would indicate that the patient
was still hypoxic (deficient in oxygen).The partial pressures of arterial oxygen and carbon
dioxide listed are within the normal range. Eupnea refers to normal respiration.

46. D. Studies have shown that showering with an antiseptic soap before surgery is the
most effective method of removing microorganisms from the skin. Shaving the site of the
intended surgery might cause breaks in the skin, thereby increasing the risk of infection;
however, if indicated, shaving, should be done immediately before surgery, not the day
before. A topical antiseptic would not remove microorganisms and would be beneficial only
after proper cleaning and rinsing. Tub bathing might transfer organisms to another body
site rather than rinse them away.

47. C. The leg muscles are the strongest muscles in the body and should bear the greatest
stress when lifting. Muscles of the abdomen, back, and upper arms may be easily injured.

48. C. The factors, known as Virchows triad, collectively predispose a patient to
thromboplebitis; impaired venous return to the heart, blood hypercoagulability, and injury
to a blood vessel wall. Increased partial thromboplastin time indicates a prolonged bleeding
time during fibrin clot formation, commonly the result of anticoagulant (heparin) therapy.
Arterial blood disorders (such as pulsus paradoxus) and lung diseases (such as COPD) do
not necessarily impede venous return of injure vessel walls.

49. A. Because of restricted respiratory movement, a recumbent, immobilize patient is at
particular risk for respiratory acidosis from poor gas exchange; atelectasis from reduced
surfactant and accumulated mucus in the bronchioles, and hypostatic pneumonia from
bacterial growth caused by stasis of mucus secretions.

50. B. The immobilized patient commonly suffers from urine retention caused by decreased
muscle tone in the perineum. This leads to bladder distention and urine stagnation, which
provide an excellent medium for bacterial growth leading to infection. Immobility also
results in more alkaline urine with excessive amounts of calcium, sodium and phosphate,
a gradual decrease in urine production, and an increased specific gravity.



















1. Jake is complaining of shortness of breath. The nurse assesses his respiratory rate to be
30 breaths per minute and documents that Jake is tachypneic. The nurse understands that
tachypnea means:

a. Pulse rate greater than 100 beats per minute
b. Blood pressure of 140/90
c. Respiratory rate greater than 20 breaths per minute
d. Frequent bowel sounds


2. The nurse listens to Mrs. Sullens lungs and notes a hissing sound or musical sound. The
nurse documents this as:

a. Wheezes
b. Rhonchi
c. Gurgles
d. Vesicular


3. The nurse in charge measures a patients temperature at 101 degrees F. What is the
equivalent centigrade temperature?

a. 36.3 degrees C
b. 37.95 degrees C
c. 40.03 degrees C
d. 38.01 degrees C


4. Which approach to problem solving tests any number of solutions until one is found that
works for that particular problem?

a. Intuition
b. Routine
c. Scientific method
d. Trial and error


5. What is the order of the nursing process?

a. Assessing, diagnosing, implementing, evaluating, planning
b. Diagnosing, assessing, planning, implementing, evaluating
c. Assessing, diagnosing, planning, implementing, evaluating
d. Planning, evaluating, diagnosing, assessing, implementing


6. During the planning phase of the nursing process, which of the following is the outcome?

a. Nursing history
b. Nursing notes
c. Nursing care plan
d. Nursing diagnosis


7. What is an example of a subjective data?

a. Heart rate of 68 beats per minute
b. Yellowish sputum
c. Client verbalized, I feel pain when urinating.
d. Noisy breathing


8. Which expected outcome is correctly written?

a. The patient will feel less nauseated in 24 hours.
b. The patient will eat the right amount of food daily.
c. The patient will identify all the high-salt food from a prepared list by discharge.
d. The patient will have enough sleep.


9. Which of the following behaviors by Nurse Jane Robles demonstrates that she
understands well th elements of effecting charting?

a. She writes in the chart using a no. 2 pencil.
b. She noted: appetite is good this afternoon.
c. She signs on the medication sheet after administering the medication.
d. She signs her charting as follow: J.R


10. What is the disadvantage of computerized documentation of the nursing process?

a. Accuracy
b. Legibility
c. Concern for privacy
d. Rapid communication


11. The theorist who believes that adaptation and manipulation of stressors are related to
foster change is:

a. Dorothea Orem
b. Sister Callista Roy
c. Imogene King
d. Virginia Henderson


12. Formulating a nursing diagnosis is a joint function of:

a. Patient and relatives
b. Nurse and patient
c. Doctor and family
d. Nurse and doctor


13. Mrs. Caperlac has been diagnosed to have hypertension since 10 years ago. Since then,
she had maintained low sodium, low fat diet, to control her blood pressure. This practice is
viewed as:

a. Cultural belief
b. Personal belief
c. Health belief
d. Superstitious belief


14. Becky is on NPO since midnight as preparation for blood test. Adreno-cortical response
is activated. Which of the following is an expected response?

a. Low blood pressure
b. Warm, dry skin
c. Decreased serum sodium levels
d. Decreased urine output


15. What nursing action is appropriate when obtaining a sterile urine specimen from an
indwelling catheter to prevent infection?

a. Use sterile gloves when obtaining urine.
b. Open the drainage bag and pour out the urine.
c. Disconnect the catheter from the tubing and get urine.
d. Aspirate urine from the tubing port using a sterile syringe.


16. A client is receiving 115 ml/hr of continuous IVF. The nurse notices that the
venipuncture site is red and swollen. Which of the following interventions would the nurse
perform first?

a. Stop the infusion
b. Call the attending physician
c. Slow that infusion to 20 ml/hr
d. Place a clod towel on the site


17. The nurse enters the room to give a prescribed medication but the patient is inside the
bathroom. What should the nurse do?

a. Leave the medication at the bedside and leave the room.
b. After few minutes, return to that patients room and do not leave until the patient takes
the medication.
c. Instruct the patient to take the medication and leave it at the bedside.
d. Wait for the patient to return to bed and just leave the medication at the bedside.


18. Which of the following is inappropriate nursing action when administering NGT feeding?

a. Place the feeding 20 inches above the pint if insertion of NGT.
b. Introduce the feeding slowly.
c. Instill 60ml of water into the NGT after feeding.
d. Assist the patient in fowlers position.


19. A female patient is being discharged after thyroidectomy. After providing the medication
teaching. The nurse asks the patient to repeat the instructions. The nurse is performing
which professional role?

a. Manager
b. Caregiver
c. Patient advocate
d. Educator


20. Which data would be of greatest concern to the nurse when completing the nursing
assessment of a 68-year-old woman hospitalized due to Pneumonia?

a. Oriented to date, time and place
b. Clear breath sounds
c. Capillary refill greater than 3 seconds and buccal cyanosis
d. Hemoglobin of 13 g/dl


21. During a change-of-shift report, it would be important for the nurse relinquishing
responsibility for care of the patient to communicate. Which of the following facts to the
nurse assuming responsibility for care of the patient?

a. That the patient verbalized, My headache is gone.
b. That the patients barium enema performed 3 days ago was negative
c. Patients NGT was removed 2 hours ago
d. Patients family came for a visit this morning.


22. Which statement is the most appropriate goal for a nursing diagnosis of diarrhea?

a. The patient will experience decreased frequency of bowel elimination.
b. The patient will take anti-diarrheal medication.
c. The patient will give a stool specimen for laboratory examinations.
d. The patient will save urine for inspection by the nurse.


23. Which of the following is the most important purpose of planning care with this patient?

a. Development of a standardized NCP.
b. Expansion of the current taxonomy of nursing diagnosis
c. Making of individualized patient care
d. Incorporation of both nursing and medical diagnoses in patient care


24. Using Maslows hierarchy of basic human needs, which of the following nursing
diagnoses has the highest priority?

a. Ineffective breathing pattern related to pain, as evidenced by shortness of breath.
b. Anxiety related to impending surgery, as evidenced by insomnia.
c. Risk of injury related to autoimmune dysfunction
d. Impaired verbal communication related to tracheostomy, as evidenced by inability to
speak.


25. When performing an abdominal examination, the patient should be in a supine position
with the head of the bed at what position?

a. 30 degrees
b. 90 degrees
c. 45 degrees
d. 0 degree

















1. (C) Respiratory rate greater than 20 breaths per minute
A respiratory rate of greater than 20 breaths per minute is tachypnea. A blood pressure of
140/90 is considered hypertension. Pulse greater than 100 beats per minute is tachycardia.
Frequent bowel sounds refer to hyper-active bowel sounds.

2. (A) Wheezes
Wheezes are indicated by continuous, lengthy, musical; heard during inspiration or
expiration. Rhonchi are usually coarse breath sounds. Gurgles are loud gurgling, bubbling
sound. Vesicular breath sounds are low pitch, soft intensity on expiration.

3. (B) 37.95 degrees C
To convert F to C use this formula, ( F 32 ) (0.55). While when converting C to F use
this formula, ( C x 1.8) + 32. Note that 0.55 is 5/9 and 1.8 is 9/5.

4. (D) Trial and error
The trial and error method of problem solving isnt systematic (as in the scientific method of
problem solving) routine, or based on inner prompting (as in the intuitive method of
problem solving).

5. (C) Assessing, diagnosing, planning, implementing, evaluating
The correct order of the nursing process is assessing, diagnosing, planning, implementing,
evaluating.

6. (C) Nursing care plan
The outcome, or the product of the planning phase of the nursing process is a Nursing care
plan.

7. (C) Client verbalized, I feel pain when urinating.
Subjective data are those that can be described only by the person experiencing it.
Therefore, only the patient can describe or verify whether he is experiencing pain or not.

8. (C) The patient will identify all the high-salt food from a prepared list by
discharge.
Expected outcomes are specific, measurable, realistic statements of goal attainment. The
phrases right amount, less nauseated and enough sleep are vague and not
measurable.

9. (C) She signs on the medication sheet after administering the medication.
A nurse should record a nursing intervention (ex. Giving medications) after performing the
nursing intervention (not before). Recording should also be done using a pen, be complete,
and signed with the nurses full name and title.

10. (C) Concern for privacy
A patients privacy may be violated if security measures arent used properly or if policies
and procedures arent in place that determines what type of information can be retrieved,
by whom, and for what purpose.

11. (B) Sister Callista Roy
Sister Roys theory is called the adaptation theory and she viewed each person as a unified
biophysical system in constant interaction with a changing environment. Orems theory is
called self-care deficit theory and is based on the belief that individual has a need for self-
care actions. Kings theory is the Goal attainment theory and described nursing as a helping
profession that assists individuals and groups in society to attain, maintain, and restore
health. Henderson introduced the nature of nursing model and identified the 14 basic needs.

12. (B) Nurse and patient
Although diagnosing is basically the nurses responsibility, input from the patient is essential
to formulate the correct nursing diagnosis.

13. (C) Health belief
Health belief of an individual influences his/her preventive health behavior.

14. (D) Decreased urine output
Adreno-cortical response involves release of aldosterone that leads to retention of sodium
and water. This results to decreased urine output.

15. (D) Aspirate urine from the tubing port using a sterile syringe.
The nurse should aspirate the urine from the port using a sterile syringe to obtain a urine
specimen. Opening a closed drainage system increase the risk of urinary tract infection.

16. (A) Stop the infusion
The sign and symptoms indicate extravasation so the IVF should be stopped immediately
and put warm not cold towel on the affected site.

17. (B) After few minutes, return to that patients room and do not leave until the
patient takes the medication
This is to verify or to make sure that the medication was taken by the patient as directed.

18. (A) Place the feeding 20 inches above the pint if insertion of NGT.
The height of the feeding is above 12 inches above the point of insertion, bot 20 inches. If
the height of feeding is too high, this results to very rapid introduction of feeding. This may
trigger nausea and vomiting.

19. (D) Educator
When teaching a patient about medications before discharge, the nurse is acting as an
educator. A caregiver provides direct care to the patient. The nurse acts as s patient
advocate when making the patients wishes known to the doctor.

20. (C) Capillary refill greater than 3 seconds and buccal cyanosis
Capillary refill greater than 3 seconds and buccal cyanosis indicate decreased oxygen to the
tissues which requires immediate attention/intervention. Oriented to date, time and place,
hemoglobin of 13 g/dl are normal data.

21. (C) Patients NGT was removed 2 hours ago
The change-of-shift report should indicate significant recent changes in the patients
condition that the nurse assuming responsibility for care of the patient will need to monitor.
The other options are not critical enough to include in the report.

22. (A) The patient will experience decreased frequency of bowel elimination.
The goal is the opposite, healthy response of the problem statement of the nursing
diagnosis. In this situation, the problem statement is diarrhea.

23. (C) Making of individualized patient care
To be effective, the nursing care plan developed in the planning phase of the nursing
process must reflect the individualized needs of the patient.

24. (A) Ineffective breathing pattern related to pain, as evidenced by shortness of
breath.
Physiologic needs (ex. Oxygen, fluids, nutrition) must be met before lower needs (such as
safety and security, love and belongingness, self-esteem and self-actualization) can be met.
Therefore, physiologic needs have the highest priority.

25. (D) 0 degree
The patient should be positioned with the head of the bed completely flattened to perform
an abdominal examination. If the head of the bed is elevated, the abdominal muscles and
organs can be bunched up, altering the findings



















1. A patient is wearing a soft wrist-safety device. Which of the following nursing assessment
is considered abnormal?

a. Palpable radial pulse
b. Palpable ulnar pulse
c. Capillary refill within 3 seconds
d. Bluish fingernails, cool and pale fingers


2. Pias serum sodium level is 150 mEq/L. Which of the following food items does the nurse
instruct Pia to avoid?

a. broccoli
b. sardines
c. cabbage
d. tomatoes


3. Jason, 3 years old vomited. His mom stated, He vomited 6 ounces of his formula this
morning. This statement is an example of:

a. objective data from a secondary source
b. objective data from a primary source
c. subjective data from a primary source
d. subjective data from a secondary source


4. Which of the following is a nursing diagnosis?

a. Hypethermia
b. Diabetes Mellitus
c. Angina
d. Chronic Renal Failure


5. What is the characteristic of the nursing process?

a. stagnant
b. inflexible
c. asystematic
d. goal-oriented


6. A skin lesion which is fluid-filled, less than 1 cm in size is called:

a. papule
b. vesicle
c. bulla
d. macule


7. During application of medication into the ear, which of the following is inappropriate
nursing action?

a. In an adult, pull the pinna upward.
b. Instill the medication directly into the tympanic membrane.
c. Warm the medication at room or body temperature.
d. Press the tragus of the ear a few times to assist flow of medication into the ear canal.


8. Which of the following is appropriate nursing intervention for a client who is grieving over
the death of her child?

a. Tell her not to cry and it will be better.
b. Provide opportunity to the client to tell their story.
c. Encourage her to accept or to replace the lost person.
d. Discourage the client in expressing her emotions.


9. It is the gradual decrease of the bodys temperature after death.

a. livor mortis
b. rigor mortis
c. algor mortis
d. none of the above


10. When performing an admission assessment on a newly admitted patient, the nurse
percusses resonance. The nurse knows that resonance heard on percussion is most
commonly heard over which organ?

a. thigh
b. liver
c. intestine
d. lung


11. The nurse is aware that Bells palsy affects which cranial nerve?

a. 2nd CN (Optic)
b. 3rd CN (Occulomotor)
c. 4th CN (Trochlear)
d. 7th CN (Facial)


12. Prolonged deficiency of Vitamin B9 leads to:

a. scurvy
b. pellagra
c. megaloblastic anemia
d. pernicious anemia


13. Nurse Cherry is teaching a 72 year old patient about a newly prescribed medication.
What could cause a geriatric patient to have difficulty retaining knowledge about the newly
prescribed medication?

a. Absence of family support
b. Decreased sensory functions
c. Patient has no interest on learning
d. Decreased plasma drug levels


14. When assessing a patients level of consciousness, which type of nursing intervention is
the nurse performing?

a. Independent
b. Dependent
c. Collaborative
d. Professional


15. Claire is admitted with a diagnosis of chronic shoulder pain. By definition, the nurse
understands that the patient has had pain for more than:

a. 3 months
b. 6 months
c. 9 months
d. 1 year


16. Which of the following statements regarding the nursing process is true?

a. It is useful on outpatient settings.
b. It progresses in separate, unrelated steps.
c. It focuses on the patient, not the nurse.
d. It provides the solution to all patient health problems.


17. Which of the following is considered significant enough to require immediate
communication to another member of the health care team?

a. Weight loss of 3 lbs in a 120 lb female patient.
b. Diminished breath sounds in patient with previously normal breath sounds
c. Patient stated, I feel less nauseated.
d. Change of heart rate from 70 to 83 beats per minute.


18. To assess the adequacy of food intake, which of the following assessment parameters is
best used?

a. food preferences
b. regularity of meal times
c. 3-day diet recall
d. eating style and habits


19. Van Fajardo is a 55 year old who was admitted to the hospital with newly diagnosed
hepatitis. The nurse is doing a patient teaching with Mr. Fajardo. What kind of role does the
nurse assume?

a. talker
b. teacher
c. thinker
d. doer


20. When providing a continuous enteral feeding, which of the following action is essential
for the nurse to do?

a. Place the client on the left side of the bed.
b. Attach the feeding bag to the current tubing.
c. Elevate the head of the bed.
d. Cold the formula before administering it.


21. Kussmauls breathing is;

a. Shallow breaths interrupted by apnea.
b. Prolonged gasping inspiration followed by a very short, usually inefficient expiration.
c. Marked rhythmic waxing and waning of respirations from very deep to very shallow
breathing and temporary apnea.
d. Increased rate and depth of respiration.


22. Presty has terminal cancer and she refuses to believe that loss is happening ans she
assumes artificial cheerfulness. What stage of grieving is she in?

a. depression
b. bargaining
c. denial
d. acceptance


23. Immunization for healthy babies and preschool children is an example of what level of
preventive health care?

a. Primary
b. Secondary
c. Tertiary
d. Curative


24. Which is an example of a subjective data?

a. Temperature of 38 0C
b. Vomiting for 3 days
c. Productive cough
d. Patient stated, My arms still hurt.


25. The nurse is assessing the endocrine system. Which organ is part of the endocrine
system?

a. Heart
b. Sinus
c. Thyroid
d. Thymus

















1. (D) Bluish fingernails, cool and pale fingers
A safety device on the wrist may impair blood circulation. Therefore, the nurse should
assess the patient for signs of impaired circulation such as bluish fingernails, cool and pale
fingers. Palpable radial and ulnar pulses, capillary refill within 3 seconds are all normal
findings.

2. (B) sardines
The normal serum sodium level is 135 to 145 mEq/L, the client is having hypernatremia. Pia
should avoid food high in sodium like processed food. Broccoli, cabbage and tomatoes are
good source of Vitamin C.

3. (A) objective data from a secondary source
Jason is the primary source; his mother is a secondary source. The data is objective
because it can be perceived by the senses, verified by another person observing the same
patient, and tested against accepted standards or norms.

4. (A) Hypethermia
Hyperthermia is a NANDA-approved nursing diagnosis. Diabetes Mellitus, Angina and
Chronic Renal Failure are medical diagnoses.

5. (D) goal-oriented
The nursing process is goal-oriented. It is also systematic, patient-centered, and dynamic.

6. (B) vesicle
Vesicle is a circumscribed circulation containing serous fluid or blood and less than 1 cm
(ex. Blister, chicken pox).

7. (B) Instill the medication directly into the tympanic membrane.
During the application of medication it is inappropriate to instill the medication directly into
the tympanic membrane. The right thing to do is instill the medication along the lateral wall
of the auditory canal.

8. (B) Provide opportunity to the client to tell their story.
Providing a grieving person an opportunity to tell their story allows the person to express
feelings. This is therapeutic in assisting the client resolve grief.

9. (C) algor mortis
Algor mortis is the decrease of the bodys temperature after death. Livor mortis is the
discoloration of the skin after death. Rigor mortis is the stiffening of the body that occurs
about 2-4 hours after death.

10. (D) lung
Resonance is loud, low-pitched and long duration thats heard most commonly over an air-
filled tissue such as a normal lung.

11. (D) 7th CN (Facial)
Bells palsy is the paralysis of the motor component of the 7th caranial nerve, resulting in
facial sag, inability to close the eyelid or the mouth, drooling, flat nasolabial fold and loss of
taste on the affected side of the face.

12. (C) megaloblastic anemia
Prolonged Vitamin B9 deficiency will lead to megaloblastic anemia while pernicious anemia
results in deficiency in Vitamin B12. Prolonged deficiency of Vitamin C leads to scurvy and
Pellagra results in deficiency in Vitamin B3.

13. (B) Decreased sensory functions
Decreased in sensory functions could cause a geriatric patient to have difficulty retaining
knowledge about the newly prescribed medications. Absence of family support and no
interest on learning may affect compliance, not knowledge retention. Decreased plasma
levels do not alter patients knowledge about the drug.

14. (A) Independent
Independent nursing interventions involve actions that nurses initiate based on their own
knowledge and skills without the direction or supervision of another member of the health
care team.

15. (B) 6 months
Chronic pain s usually defined as pain lasting longer than 6 months.

16. (C) It focuses on the patient, not the nurse.
The nursing process is patient-centered, not nurse-centered. It can be use in any setting,
and the steps are related. The nursing process cant solve all patient health problems.

17. (B) Diminished breath sounds in patient with previously normal breath sounds
Diminished breath sound is a life threatening problem therefore it is highly priority because
they pose the greatest threat to the patients well-being.

18. (C) 3-day diet recall
3-day diet recall is an example of dietary history. This is used to indicate the adequacy of
food intake of the client.

19. (B) teacher
The nurse will assume the role of a teacher in this therapeutic relationship. The other roles
are inappropriate in this situation.

20. (C) Elevate the head of the bed.
Elevating the head of the bed during an enteral feeding prevents aspiration. The patient
may be placed on the right side to prevent aspiration. Enteral feedings are given at room
temperature to lessen GI distress. The enteral tubing should be changed every 24 hours to
limit microbial growth.

21. (D) Increased rate and depth of respiration.
Kussmaul breathing is also called as hyperventilation. Seen in metabolic acidosis and renal
failure. Option A refers to Biots breathing. Option B is apneustic breathing and option C is
the Cheyne-stokes breathing.

22. (C) denial
The client is in denial stage because she is unready to face the reality that loss is happening
and she assumes artificial cheerfulness.

23. (A) Primary
The primary level focuses on health promotion. Secondary level focuses on health
maintenance. Tertiary focuses on rehabilitation. There is n Curative level of preventive
health care problems.

24. (D) Patient stated, My arms still hurt.
Subjective data are apparent only to the person affected and can or verified only by that
person.

25. (C) Thyroid
The thyroid is part of the endocrine system. Heart, sinus and thymus are not.





















1. Nurse Brenda is teaching a patient about a newly prescribed drug. What could cause a
geriatric patient to have difficulty retaining knowledge about prescribed medications?

a. Decreased plasma drug levels
b. Sensory deficits
c. Lack of family support
d. History of Tourette syndrome


2. When examining a patient with abdominal pain the nurse in charge should assess:

a. Any quadrant first
b. The symptomatic quadrant first
c. The symptomatic quadrant last
d. The symptomatic quadrant either second or third


3. The nurse is assessing a postoperative adult patient. Which of the following should the
nurse document as subjective data?

a. Vital signs
b. Laboratory test result
c. Patients description of pain
d. Electrocardiographic (ECG) waveforms


4. A male patient has a soft wrist-safety device. Which assessment finding should the nurse
consider abnormal?

a. A palpable radial pulse
b. A palpable ulnar pulse
c. Cool, pale fingers
d. Pink nail beds


5. Which of the following planes divides the body longitudinally into anterior and posterior
regions?

a. Frontal plane
b. Sagittal plane
c. Midsagittal plane
d. Transverse plane


6. A female patient with a terminal illness is in denial. Indicators of denial include:

a. Shock dismay
b. Numbness
c. Stoicism
d. Preparatory grief


7. The nurse in charge is transferring a patient from the bed to a chair. Which action does
the nurse take during this patient transfer?

a. Position the head of the bed flat
b. Helps the patient dangle the legs
c. Stands behind the patient
d. Places the chair facing away from the bed


8. A female patient who speaks a little English has emergency gallbladder surgery, during
discharge preparation, which nursing action would best help this patient understand wound
care instruction?

a. Asking frequently if the patient understands the instruction
b. Asking an interpreter to replay the instructions to the patient.
c. Writing out the instructions and having a family member read them to the patient
d. Demonstrating the procedure and having the patient return the demonstration


9. Before administering the evening dose of a prescribed medication, the nurse on the
evening shift finds an unlabeled, filled syringe in the patients medication drawer. What
should the nurse in charge do?

a. Discard the syringe to avoid a medication error
b. Obtain a label for the syringe from the pharmacy
c. Use the syringe because it looks like it contains the same medication the nurse was
prepared to give
d. Call the day nurse to verify the contents of the syringe


10. When administering drug therapy to a male geriatric patient, the nurse must stay
especially alert for adverse effects. Which factor makes geriatric patients to adverse drug
effects?

a. Faster drug clearance
b. Aging-related physiological changes
c. Increased amount of neurons
d. Enhanced blood flow to the GI tract


11. A female patient is being discharged after cataract surgery. After providing medication
teaching, the nurse asks the patient to repeat the instructions. The nurse is performing
which professional role?

a. Manager
b. Educator
c. Caregiver
d. Patient advocate


12. A female patient exhibits signs of heightened anxiety. Which response by the nurse is
most likely to reduce the patients anxiety?

a. Everything will be fine. Dont worry.
b. Read this manual and then ask me any questions you may have.
c. Why dont you listen to the radio?
d. Lets talk about whats bothering you.


13. A scrub nurse in the operating room has which responsibility?

a. Positioning the patient
b. Assisting with gowning and gloving
c. Handling surgical instruments to the surgeon
d. Applying surgical drapes


14. A patient is in the bathroom when the nurse enters to give a prescribed medication.
What should the nurse in charge do?

a. Leave the medication at the patients bedside
b. Tell the patient to be sure to take the medication. And then leave it at the bedside
c. Return shortly to the patients room and remain there until the patient takes the
medication
d. Wait for the patient to return to bed, and then leave the medication at the bedside


15. The physician orders heparin, 7,500 units, to be administered subcutaneously every 6
hours. The vial reads 10,000 units per millilitre. The nurse should anticipate giving how
much heparin for each dose?

a. ml
b. ml
c. ml
d. 1 ml


16. The nurse in charge measures a patients temperature at 102 degrees F. what is the
equivalent Centigrade temperature?

a. 39 degrees C
b. 47 degrees C
c. 38.9 degrees C
d. 40.1 degrees C


17. To evaluate a patient for hypoxia, the physician is most likely to order which laboratory
test?

a. Red blood cell count
b. Sputum culture
c. Total hemoglobin
d. Arterial blood gas (ABG) analysis


18. The nurse uses a stethoscope to auscultate a male patients chest. Which statement
about a stethoscope with a bell and diaphragm is true?

a. The bell detects high-pitched sounds best
b. The diaphragm detects high-pitched sounds best
c. The bell detects thrills best
d. The diaphragm detects low-pitched sounds best


19. A male patient is to be discharged with a prescription for an analgesic that is a
controlled substance. During discharge teaching, the nurse should explain that the patient
must fill this prescription how soon after the date on which it was written?

a. Within 1 month
b. Within 3 months
c. Within 6 months
d. Within 12 months


20. Which human element considered by the nurse in charge during assessment can affect
drug administration?

a. The patients ability to recover
b. The patients occupational hazards
c. The patients socioeconomic status
d. The patients cognitive abilities


21. When explaining the initiation of I.V. therapy to a 2-year-old child, the nurse should:

a. Ask the child, Do you want me to start the I.V. now?
b. Give simple directions shortly before the I.V. therapy is to start
c. Tell the child, This treatment is for your own good
d. Inform the child that the needle will be in place for 10 days


22. All of the following parts of the syringe are sterile except the:

a. Barrel
b. Inside of the plunger
c. Needle tip
d. Barrel tip


23. The best way to instill eye drops is to:

a. Instruct the patient to lock upward, and drop the medication into the center of the lower
lid
b. Instruct the patient to look ahead, and drop the medication into the center of the lower
lid
c. Drop the medication into the inner canthus regardless of eye position
d. Drop the medication into the center of the canthus regardless of eye position


24. The difference between an 18G needle and a 25G needle is the needles:

a. Length
b. Bevel angle
c. Thickness
d. Sharpness


25. A patient receiving an anticoagulant should be assessed for signs of:

a. Hypotension
b. Hypertension
c. An elevated hemoglobin count
d. An increased number of erythrocytes















1. (B) Sensory deficits
Sensory deficits could cause a geriatric patient to have difficulty retaining knowledge about
prescribed medications. Decreased plasma drug levels do not alter the patients knowledge
about the drug. A lack of family support may affect compliance, not knowledge retention.
Toilette syndrome is unrelated to knowledge retention.

2. (C) The symptomatic quadrant last
The nurse should systematically assess all areas of the abdomen, if time and the patients
condition permit, concluding with the symptomatic area. Otherwise, the nurse may elicit
pain in the symptomatic area, causing the muscles in other areas to tighten. This would
interfere with further assessment.

3. (C) Patients description of pain
Subjective data come directly from the patient and usually are recorded as direct quotations
that reflect the patients opinions or feelings about a situation. Vital signs, laboratory test
result, and ECG waveforms are examples of objective data.

4. (C) Cool, pale fingers
A safety device on the wrist may impair circulation and restrict blood supply to body tissues.
Therefore, the nurse should assess the patient for signs of impaired circulation, such as
cool, pale fingers. A palpable radial or lunar pulse and pink nail beds are normal findings.

5. (A) Frontal plane
Frontal or coronal plane runs longitudinally at a right angle to a sagittal plane dividing the
body in anterior and posterior regions. A sagittal plane runs longitudinally dividing the body
into right and left regions; if exactly midline, it is called a midsagittal plane. A transverse
plane runs horizontally at a right angle to the vertical axis, dividing the structure into
superior and inferior regions.

6. (A) Shock dismay
Shock and dismay are early signs of denial-the first stage of grief. The other options are
associated with depressiona later stage of grief.

7. (B) Helps the patient dangle the legs
After placing the patient in high Fowlers position and moving the patient to the side of the
bed, the nurse helps the patient sit on the edge of the bed and dangle the legs; the nurse
then faces the patient and places the chair next to and facing the head of the bed.

8. (D) Demonstrating the procedure and having the patient return the
demonstration
Demonstrating by the nurse with a return demonstration by the patient ensures that the
patient can perform wound care correctly. Patients may claim to understand discharge
instruction when they do not. An interpreter of family member may communicate verbal or
written instructions inaccurately.

9. (A) Discard the syringe to avoid a medication error
As a safety precaution, the nurse should discard an unlabeled syringe that contains
medication. The other options are considered unsafe because they promote error.

10. (B) Aging-related physiological changes
Aging-related physiological changes account for the increased frequency of adverse drug
reactions in geriatric patients. Renal and hepatic changes cause drugs to clear more slowly
in these patients. With increasing age, neurons are lost and blood flow to the GI tract
decreases.

11. (B) Educator
When teaching a patient about medications before discharge, the nurse is acting as an
educator. The nurse acts as a manager when performing such activities as scheduling and
making patient care assignments. The nurse performs the care giving role when providing
direct care, including bathing patients and administering medications and prescribed
treatments. The nurse acts as a patient advocate when making the patients wishes known
to the doctor.

12. (D) Lets talk about whats bothering you.
Anxiety may result from feeling of helplessness, isolation, or insecurity. This response helps
reduce anxiety by encouraging the patient to express feelings. The nurse should be
supportive and develop goals together with the patient to give the patient some control over
an anxiety-inducing situation. Because the other options ignore the patients feeling and
block communication, they would not reduce anxiety.

13. (C) Handling surgical instruments to the surgeon
The scrub nurse assist the surgeon by providing appropriate surgical instruments and
supplies, maintaining strict surgical asepsis and, with the circulating nurse, accounting for
all gauze, sponges, needles, and instruments. The circulating nurse assists the surgeon and
scrub nurse, positions the patient, applies appropriate equipment and surgical drapes,
assists with gowning and gloving, and provides the surgeon and scrub nurse with supplies.

14. (C) Return shortly to the patients room and remain there until the patient
takes the medication
The nurse should return shortly to the patients room and remain there until the patient
takes the medication to verify that it was taken as directed. The nurse should never leave
medication at the patients bedside unless specifically requested to do so.

15. (C) ml
The nurse solves the problem as follows: 10,000 units/7,500 units = 1 ml/X 10,000 X =
7,500 X= 7,500/10,000 or ml

16. (C) 38.9 degrees C
To convert Fahrenheit degrees to centigrade, use this formula: C degrees = (F degrees
32) x 5/9 C degrees = (102 32) 5/9 + 70 x 5/9 38.9 degrees C

17. (D) Arterial blood gas (ABG) analysis
All of these test help evaluate a patient with respiratory problems. However, ABG analysis is
the only test evaluates gas exchange in the lungs, providing information about patients
oxygenation status.

18. (B) The diaphragm detects high-pitched sounds best
The diaphragm of a stethoscope detects high-pitched sound best; the bell detects low
pitched sounds best. Palpation detects thrills best.

19. (C) Within 6 months
In most cases, an outpatient must fill a prescription for a controlled substance within 6
months of the date on which the prescription was written.

20. (D) The patients cognitive abilities
The nurse must consider the patients cognitive abilities to understand drug instructions. If
not, the nurse must find a family member or significant other to take on the responsibility of
administering medications in the home setting. The patients ability to recover, occupational
hazards, and socioeconomic status do not affect drug administration.

21. (B) Give simple directions shortly before the I.V. therapy is to start
Because a 2-year-old child has limited understanding, the nurse should give simple
directions and explanations of what will occur shortly before the procedure. She should try
to avoid frightening the child with the explanation and allow the child to make simple
choices, such as choosing the I.V. insertion site, if possible. However, she shouldnt ask the
child if he wants the therapy, because the answer may be No! Telling the child that the
treatment is for his own good is ineffective because a 2-year-old perceives pain as a
negative sensation and cannot understand that a painful procedure can have position
results. Telling the child how long the therapy will last is ineffective because the 2-year-old
doesnt have a good understanding of time.

22. (A) Barrel
All syringes have three parts: a tip, which connects the needle to the syringe; a barrel, the
outer part on which the measurement scales are printed; and a plunger, which fits inside
the barrel to expel the medication. The external part of the barrel and the plunger and
(flange) must be handled during the preparation and administration of the injection.
However, the inside and trip of the barrel, the inside (shaft) of the plunger, and the needle
tip must remain sterile until after the injection.

23. (A) Instruct the patient to lock upward, and drop the medication into the
center of the lower lid
Having the patient look upward reduces blinking and protects the cornea. Instilling drops in
the center of the lower lid promotes absorption because the drops are less likely to run into
the nasolacrimal duct or out of the eye.

24. (C) Thickness
Gauge is a measure of the needles thickness: The higher the number the thinner the shaft.
Therefore, an 18G needle is considerably thicker than a 25G needle.

25. (A) Hypotension
A major side effect of anticoagulant therapy is bleeding, which can be identified by
hypotension (a systolic blood pressure under 100 mm Hg). Anticoagulants do not result in
the other three conditions.

Você também pode gostar